صفحه 1:
Pulmonology: OSCE &
PMP
صفحه 2:
خانم 48 ساله که 3 ساعت پس از سقوط از پله منزل دچار تگی نفس و هذیان گویی شده و در بررسی های اولیه شواهد
بالا به دست آمده است؟
صفحه 3:
48-YEAR-OLD MAN WHO HAD SUSTAINED A TRAUMATIC FRACTURE OF
the left femoral shaft in a motorcycle accident 20 hours earlier was referred
for a deterioration of consciousness that began after the accident. He was
unresponsive to verbal stimuli, and no focal neurologic abnormalities were found
on examination. An arterial blood gas analysis revealed mild hypoxemia, but a ra-
diograph of the chest was normal. Tracheal intubation was performed for airway
protection. Magnetic resonance imaging of the brain revealed multiple hyperin-
tense punctate lesions disseminated throughout the cerebral white matter on T,-
weighted axial images (Panel A) and a so-called starfield pattern on diffusion-
weighted images (Panel B). Seven hours after presentation, widespread petechiae
developed on the chest, upper arm, and armpit (Panel C). The combination of a
recent fracture of a long bone, petechial rash, mild hypoxemia, and profound im-
pairment of consciousness prompted a diagnosis of fat embolism syndrome. The
patient recuperated gradually after orthopedic repair of the fracture and rehabilita-
tion therapy.
صفحه 4:
Ficune 2A 1Syearokd mam at 2 days after &
ttn Top, HRGT son hn a nes gm
reveal 1 predominantly peripheral یاس اه ساسا
Opectiestsneated with smooth ard nodhlar cepa theketing
‘oan, B: HRC obtained at lower level shows retve spring
Of some secondary lobules, Findings i the paint ore mar
ponomced Hin the ebscrved in the patent in Figure 1
Ficune 4. A 27-year-old man with hyposemia and buccal
petechiae 3 days aller injury. Top, A: HCE obtained just below
the tracheal hifureation reveals a predominantly nodular pattern
Note that bronchowaseular bundles are thin and smooth. Bottom,
B:TIRCT obtained at the level of pulimonary veins shows similar
pattem and severity of findings
صفحه 5:
CASE 1:
A 65 y,o man with a history of 40 p/y smoking presented because of 4 Ws of
progressive exertional dyspnea and non productive cough!!!
صفحه 6:
CASE 3:
a 67 y,oman with progressive 7 2 since 2 y
۱
Figure 6.3. HRCT scan in early phase of IPF shows predominantly subpleural reticular pattem (a), and in the late phase predominantly subpleural coarse reticular pattem, septal thick-
ening, traction bronchiectasis, and honeycombing. The ground-glass opacities are usually limited, and if extensive (>30% of the lung involved) an alternative diagnosis should be con-
sidered. This particular 70-year old patient experienced progressive dyspnea during lst 8 months, quitted smoking ost weight and was examined for suspected IPF. As he fulfilled
the major and minor criteria for IP, including the HRCT finding, the open lung biopsy was not considered necessary. As the disease progressed the corticosteroids and azathioprine
were applied with insignificant efficacy (b).
صفحه 7:
CASE 4:
A 40 y,o nonsmoker woman with progressive dyspnea of
4 m duration accompanied with fever, elevated LDH and
3
صفحه 8:
صفحه 9:
CASE 5:
A 85 y,o man with acute progressive shortness of breath
since 48 h ago
صفحه 10:
CASE 6: A 81/Y,0 MAN WITH FLU-LIKE SYMPTOMES SINCE 3 WEEKS AGO +
۳۹ ام DYSENEA
ae microscopic views of COP.
“./. Note the absence of old
» fibrosis or architectural
1g appearance &
In tissue plugs
inflammatory infiltrate.
something that is
صفحه 11:
هر دو بیمار آقای 57 ساله سیگاری 30 0/۷ با شکایت سرفه و تنگی نفس از 5 ماه قبل
صوق و كفيس
درمان؟؟
Langerhans cell histiocytosis. (A) The characteristic combination of thin-walled cysts and poorly defined nodules, some of which are just
beginning to cavitate.* (B) Image from a patient with more advanced disease. There are numerous irregularly shaped cysts bilaterally.
صفحه 12:
صفحه 13:
cunt 9.62 امعم
Diffuse bilateral thin-walled cysts throughout the lungs
in women of childbearing age are diagnostic of LAM.
Extrathoracic related findings are present in >70% of
cases.
صفحه 14:
خانم ۳۴ ساله با علایم تب و لرز و میالژی از ۱۴ روز قبل که طی ۱۲ ساعت
گذشته دچار تنگی نفس پیشرونده و هیپوکسی شدید و سیانوز شده است
صفحه 15:
۱۱۱۱3۸ 0۱۸6۱05۲16 8118184 ARDS
Chest Absence of Left
|Severity: Oxygenation _Onset_Radiograph___Atrial Hypertension _
Mild: 200 mmHg Acute Bilateral alveolar. PCWP <18 mmHg or
< Pao /Fia, < 300 mmHg or interstitial no clinical evidence
Moderate: 100 mmHg infiltrates of increased left atrial
< Pao,/Fio, > 200 mmHg pressure
Severe: Pao_/Fio,
> 100 mmHg
Abbreviations: ARDS, acute respiratory distress syndrome; Fio,, inspired O, percentage;
يدوم arterial partial pressure of O,; PCWP, pulmonary capillary wedge pressure.
صفحه 16:
InrriaL MANAGEMENT OF ARDS LUTEP739 EVIDENCE-BASED RECOMMENDATIONS FOR ARDS THERAPIES:
[Goaisendtimis: == —satment__Recommendation’ _
Thumalpressuesiied > Wal vokme <6 mig PEW eee A
6 Plateau pressure < 30 omHO lume
= RR < 35 bpm Minimized left atria filing pressures 8
High PEEP or‘open lung’ c
ديمع Prone position c
PEEP > 10 ماني Recruitment maneuvers c
ees High-ftequency ventiation D
ECMO ۴
557 Early neuromuscular blockade A
Minimize acidosis AR < 35 bpm Glucocorticoid treatment D
Surfactant replacement, Inhaled NO, Inhaled D
epoprostendl, and other antinflammatory
7 therapy (eg, ketoconazole PGE!, NSAIDs)
265 mmHg
“key:A recommended thetapy bated on ston cna evidence fom randomized
رد رم nical tials; 8, recommended therapy based on supportive but limited clinical data; C,
recommended only as alternative therapy on the basis of indeterminate evidence, D, not
recommended on the bass of clinical evidence against efficacy of therapy,
IGURE 322-5 Algorithm for the initial management of ARDS.
Clinical trials have provided evidence-based therapeutic goals fora و ی nic cade NeAlos nowsetol airman dae PEEP
stepwise approach to the early mechanical ventilation, oxygenation, positive end-expiratory pressure, PCE, prostaglandin E
and correction of acidosis and diuresis of criticaly ill patients with
ARDS. Fo, inspired O, percentage; MAP, mean arterial pressure; PBW,
predicted body weight; PEEP, positive end expiratory pressure; RR,
respiratory rate; SpO,, arterial oxyhemoglobin saturation measured by
pulse oximetry. 7
صفحه 17:
خانم جوان با شکایت درد قفسه سینه از صبح روز مراجعه با انتشار به بازوی راست و تنگی نفس.
بیمار سابقه فشار خون و 6۴۲ و سرفه مزمن را نیز داشته است.
صفحه 18:
PNEUMOMEDIASTINUM
In this condition, there is(gas in the interstices of the mediastinum.
The three main causes are (1) alveolar rupture with dissection of air
into the mediastinum; (2) perforation or rupture of the esophagus,
trachea, or main bronchi; and (3) dissection of air from the neck or
the abdomen into the mediastinum. Typically, there is severe subster-
nal chest pain with or without radiation into the neck and arms. The
physical examination usually reveals subcutaneous emphysema in the
suprasternal notch and! Hamman’s sign, which is a crunching or click-
ing noise synchronous with the heartbeat and i is best heard in the left
¢ rmed with the chest
ph.
will be absorbed faster if the patient inspires high ‘concentrations of
oxygen. If mediastinal structures are compressed, the compression can
erelieved with needle aspiration.
صفحه 19:
نام دو دستگاه مشاهده شده را بگویید و برای هر مورد یک مثال دارویی بزنید:
صفحه 20:
o% i 3
MDI with Spacer املا Diskus Handihaler Twisthaler_ Autohaler
3 > ١ BI
۱
سم 1 ©
Aerolizer Flexhaler Neohaler Respimat Pressair Turbuhaler
Ellipta
صفحه 21:
301606066066156 76
25mg/2 Smt nebule
smg/2smLreaiie
‘stata
‘SALGUTAMOL WITH IPRATROPIUM
Nebuter olson
saibuera 25 me +
‘erro 500 meg 2.5 mL
‘NEDOCROWIL Acoso whaler
لد
‘Te
Subsidised Inhalers for Asthma or COPD New Zealand March 2016
- سس rr
IPRATROPIUM Aerosol inhaler 5 8 en Aerosol inhaler
و سیم ei see
wal ee
۳ هه جع
ص امت
e = =a &
= = =
‘Mast Cell Stabilisers
سس nen اند SEER Areal ae
ae covet
عمج ee
SABA —Short-Acting Beta2 Agonists
ne awn
0
a f
=
2 ع ع
SAIRUTAMOL Aerosol nher|
هوم مهد
صفحه 22:
Table 1: Guide to addition of therapies*
Lapay Ics)
“ed boves win crosses neste cases of topes tat should not be used togeter SABA SAMA LAMA LABA AEX IKEA
مجعم ,ممم "ماده مدای
۱
ipratropium bromide (Atrovent)
‘Totrepium bromige (Spiiva™)
{lycopyrrenium bromide (Secbr™) mediate (ineuse™).
Saimeterol (Serevent)
وعم سحي . Forale™) اس ام
0 ا
bromide (URibro”) Gree)
{+ Futeasone propirata/Saimeterl (See) py scaone =
1 Busesonice/efarmoreal (Symbenrt™) Fh furoateyviantere (Bre0™)
‘Acidnium bromide (Bretarie™)
Comes
sana
ann
aa
casa
Bat!
Ics,
4
td
صفحه 23:
45 ساله با تتگی نفس و هموپتزی مراجعه نموده است. با توجه به تگرافی بیمار 5
تشخیص افتراقی مطرح نمایید:
صفحه 24:
| = infection
pulmonary abscess
TB, fungal, staph
aureus
T = trauma
Pneumatocoeles
Y = youth
CPAM
pulmonary
sequestration
bronchogenic cyst
CAVITY mnemonic :
C = cancer
bronchogenic carcinoma :
most frequently SCC
cavitatory metastasis :
again most frequently SCC
A = autoimmune granulomas
Wegener's granulomatosis
rheumatoid
arthritis (rheumatoid nodules)
V = vascular
pulmonary embolus /
pulmonary infarction
صفحه 25:
خانم ۵۶ساله با درد مفاصل از ۴سال قبل و خشکی صبحگاهی در حد ۳۰دقیقه كه از "ماه قبل دچار
تنگی نفس شده است .
۱.توصیف؟
۲.تشخیص؟؟
صفحه 26:
اور یت وزیا
Pleural effusion or thickening
Interstitial fibrosis (most frequently usual interstitial
pneumonia type)
Constrictive obliterative bronchiolitis a
Bronchiectasis
Organizing pneumonia
Follicular bronchiolitis
Drug-induced lung disease (methotrexate) |
Necrobiotic nodules/Caplan’s syndrome ’ ۱
Rheumatoid disease. Two cavitating necrobiotic nodules are
visible*
صفحه 27:
صفحه 28:
Differential Diagnosis
¢ Malignancy
* Granulomatous disease
¢ Inflammation
¢ Benign neoplasm
* Congenital
multiple nodules on left (all
less than 3 cm) and nodules
and one mass on the right
(3.4 cm)
صفحه 29:
خانم 44 ساله با شکایت تنگی نفس
مراجعه نموده و به علت رویت پلورال
افیوژن تحت آسپیراسیون مایع پلور قرار
گرفته است, تب نداشته و مایع بدبو
تق فرص 8
3 علت اصلی ایجاد این حالت؟
صفحه 30:
Lymphomas
Bronchogenic carcinoma
Other solid tumors”
Surgical (especially thoracic surgery)
Non surgical trauma
Congenital
Other unidentifiable causes
Sarcoidosis!
Congestive cardiac failure
Lymphengioleiomyomatosis (LAM)
Lymphengiomatosis
Yellow nail syndrome
Gorham’s syndrome
Castleman’s disease
Superior vena cava obstruction
Kaposi sarcoma in acquired immunodeficieney
syndrome (ATDS)
Radiation therapy to the mediastinum
Tuberculosis, histoplasmosis, or filariasis
Malignancy
Trauma
Idiopathic
Miscellaneous
Infections
Chylothorax A chylothorax occurs when the thoracic duct is disrupted
and chyle accumulates in the pleural space. The most common cause
of chylothorax is trauma (most frequently thoracic surgery), but it also
may result from tumors in the mediastinum. Patients with chylothorax
present with dyspnea, and a large pleural effusion is present on the
chest radiograph. Thoracentesis reveals milly fluid, and biochemical
analysis reveals a triglyceride level that exceeds 1.2 mmol/l. (110 mg/
dL). Patients with chylothorax and no obvious trauma should have a
lymphangiogram and a mediastinal CT scan to assess the mediastinum
for lymph nodes. The treatment of choice for most chylothoraxes is
insertion of a chest tube plus the administration of octreotide. If these
modalities fail, a pleuroperitoneal shunt should be placed unless the
patient has chylous ascites. Alternative treatments are ligation of the
thoracic duct and percutaneous transabdominal thoracic duct block-
age. Patients with chylothoraxes should not undergo prolonged tube
thoracostomy with chest tube drainage because this will lead to mal-
nutrition and immunologic incompetence.
صفحه 31:
آقای ۶۰ ساله سیگاری با تنگی نفس از ۵ ماه
قبل که از ۲ ماه قبل علایم وی تشدید یافته است.
توصيف؟ 1
تشخيص؟؟
درمان؟؟؟
صفحه 32:
FIGURE 5-12, Usual interstitial pneumonia (UIP). A, There are coarse rekular ntrstal markings representing fibrosis, seen herein a dose-up of the right lng base (whe crcl). The findings in UP
‘occur predominantly at the lung bases, B, An axal CT scan ofthe chest shows abnormalities atthe lung bases ina subpleural location, the typical dstnbution for UP. These are small estic spaces called fon
‘excombing (whe arrows) with evidence of bronchiectass a5 manifest by thickened bronchial wals (dotted white aro)
صفحه 33:
Usual interstitial pneumonia. In the upper lobes anteriorly
there are peripheral irregular lines with areas of
honeycombing.*
Usual interstitial pneumonia. HRCT abnormalities predominate in
the posterior, and subpleural regions of the lower lobes and
comprise honeycombing and traction bronchiectasis within the
abnormal lung.*
صفحه 34:
آقای ۴۴ ساله سیگاری با سرفه و تنگی نفس فعالیتی از ۳ ماه قبل:
توص یف
#تشخيص و تشخيصن. اقتراف؟؟
۳. درمان؟؟؟
صفحه 35:
RB-ILD. HRCT shows:
(A) subtle areas of ground
glass opacification and
(B) ill-defined centrilobular
nodules *
DIP. There is
diffuse ground-
glass opacification
throughout the
lungs due to
inflammatory
> infitrate.!
صفحه 36:
آقای ۳۲ ساله با آسم مقاوم به درمان از ۱ سال قبل که اخیرا حملات وی بسیار شدید شده
7”
صفحه 37:
صفحه 38:
~ f FINGER-IN-GLOVE SIGN
صفحه 39:
خانم ۳۴ ساله با یک نوبت سابقه پنوموتوراکس سال قبل با شکایت تنگی نفس ناگهانی و
سی تی اسکن زیر به شما مراجعه نموده است. توصیف و تشخیص و درمان؟؟؟
صفحه 40:
Treatment Progression is common, with a median survival of 8-10
years from diagnosis. No therapy is of proven benefit in LAM.
Sirolimus, an inhibitor of the mammalian target of rapamycin (mTOR),
appears to be an active agent for LAM. After 12 months, it stabilized
ung function (FVG, FEV, and functional residual capacity) and was
associated with a reduction in symptoms and improvement in quality
of life. Adverse effects (c.g., mucositis, diarrhea, nausea, hypercholes-
terolemia, acneiform rash, peripheral edema) were more common in
the sirolimus group, but serious adverse effects were not increased.
Subjects were followed off sirolimus for an additional 12 months,
during which time pulmonary function declined at the same rate as in
the placebo group. Progesterone and luteinizing hormone-releasing
hormone analogues have been used. Oophorectomy is no longer rec-
ommended, and estrogen-containing drugs should be discontinued.
Lung transplantation offers the only hope for cure despite reports of
recurrent disease in the transplanted lung,
PULMONARY LYMPHANGIOLEIOMYOMATOSIS
i || Manifestations Pulmonary LAM is a rare condition that afflicts
premenopausal women and should be suspected in young women
with “emphysema,” recurrent pneumothorax, or chylous pleural
effusion. It is often misdiagnosed as asthma or chronic obstructive
pulmonary disease, Whites are affected much more commonly than
are members of other racial groups. The disease accelerates during
pregnancy and abates after oophorectomy. Common complaints at
presentation are dyspnea, cough, and chest pain, Hemoptysis may
be life threatening. Spontaneous pneumothorax occurs in 50% of
patients; it may be bilateral and necessitate pleurodesis. Meningioma
and renal angiomyolipomas (hamartomas), characteristic findings in
the genetic disorder tuberous sclerosis, are also common in patients
with LAM. Chylothorax, chyloperitoneum (chylous ascites), chyluria,
and chylopericardium are other complications, Pulmonary function
testing usually reveals an obstructive or mixed obstructive-restrictive
pattern, and gas exchange is often abnormal. HRCT shows thin-walled
‘cysts surrounded by normal lung without zonal predominance,
Histologic Findings Pathologically, LAM is characterized by the pro-
liferation of atypical pulmonary interstitial smooth muscle and cyst
formation. The immature-appearing smooth-muscle cells react with
monoclonal antibody HMB45, which recognizes a 100-kDa glycopro-
tein (gp100) originally found in human melanoma cells.
صفحه 41:
آقای ۵۳ ساله مسیول تاسیسات بیمارستان که با شکایت تنگی نفس و سرفه های
خشک از ۳ ماه قبل مراجعه نموده است.
۱ تشخیص و توصیف؟
۲ درمان پیشنهادی؟؟
صفحه 42:
Asbestosis
Radiographic features
Plain radiograph
There are no pathognomonic radiological features specific for asbestosis '. Chest
radiograph may show irregular opacities with a fine reticular pattern, Additional
evidence of asbestos exposure such as calcified or noncalcified pleural plaques may be
evident.
Histology from a lung biopsy showing asbestos bodies. Ferruginous bodies consisting of asbestos fibers coated by iton-protein-mucopelysaccharde material with
Jgolden brown, beaded appearance. Th two longest asbestos bodies a the center ofthe igure are present within a muhinuceated giant cll. Hematoxylin and eosin stain,
+400), (Courtesy of Dr. David Hwang, Toronto General Hospital)
صفحه 43:
آقای ۳۳۴ ساله سیگاری با شکایت سرفه» تنگی نفس و کاهش وزن ۱۲ کیلو از ۶ ماه قبل که
از ۴ هفته قبل دچار پلی اوری نیز شده است.
۱.توصیف و تشخیص؟
۲.درمان؟؟
صفحه 44:
These cysts start as round structures but finally coalesce to be-
come the typical bizarre shaped cysts of LCH.
In patients with LCH 95% have a smoking history.
On the left radiological pathological correlation of Langerhans
cell histiocytosis in respectively nodular stage and early and
late cystic stage
صفحه 45:
خانم ۴۴ ساله با سرفه خلط دار کف آلود از ۲ ماه قبل همراه با ضعف و کاهش وزن و
تنگی نفس فعالیتی:
صفحه 46:
‘PULMONARY ALVEOLAR PROTEINOSIS (PAP)
inal Manifestations Although not strictly an TLD, PAP resembles
ais therefore considered with these condition, It hasbeen p
hata defect In macrophage function
ability to process surfactant, may play
of PAP: acquired (>90% of all case),
congenital, and seo ital PAP is transmitted in an ato
imal recesive manner and is caused by homorygosty for a frame
‘hifi mutation (121ins2) in the SP-B gene, which leads to an unstable
SP-B mRNA, reduced protein levels, and secondary disturbances of
SP-C processing. Secondary PAP is rare among adults and is caused
by ysnurie protein intolerance, acute alcoss and other inhalato
gradromes, immunodeficiency disorders, and malignancies (almost
exclusively of hematopoietic origin) and hematopoietic disorders,
nonproductive cough 1s common, But occasionally expec
toration of “chunky” gelatinous material may occur. Palyythemia
hypergammaglobulinemia, and increased LDH level are-common,
Markesly elevated serum levels of lung surfactant proteins A and D
have been found in PAP. In the absence of any known secondary cause
‘of PAP, an elevated serum anti-GM-CSF ter is highly sensitive and
specific forthe diagnosis of acquired PAP. BAL. id levels of ant
GM-CSF antibodies correlate bette withthe severity of PAP than do
serum titers, Radiographically bilateral symmetric alveolar opacities
located centrally in mide and lower lung zones result ina “bat wing
bution. HRCT shows a ground-gass opacification and thickened
Intralobular structure and interlobular septa.
00
i Findings This difuse dis
tion of an amorphous, per
is characterized by the accu
ic aid-Schif-postive lipopo whole lung lavage
tinaceous material inthe distal ar paces. There a litle or no hung
inflammation, and the underlying hung architecture preserved
(SSE pede وش وت
صفحه 47:
آقای ۵۰ ساله پا سابقه
ترومای شدید و بستری ۴
ماهه در بخش مراقبتهای ویژه ۳ es =e
ع هد 3 2 3
که با شکایت تنقس صدا دار به ve 7 سب
5 8 0 340 33
شما مراجعه نموده است. * 37 0
6 كيه 000
3 5 7
00 3 ات
ico (bl) و 6
$0 98
SSPRED = percentage of praised va
3
مگ
:
ee
5 Expired
مسا ۰
و 5
a2
2
صفحه 48:
‘PRED
3
134
148
38
sé
1
3
لم8
ذا عناص
inspired 5
37
a
oat
340.
۳
کف
3
7
838
me
av
RY/TLC
ve
FEY,
FEV/FVe
Mw
مت
ico (bbs)
Spo:
SERED pceing: priced ae,
Exp flow (Lise)
Insp fow
Interpretation: Abnormal, Severe fixed airway obstruction is indicated by the reduced FEV, and MVV and shape of the inspiratory and expiratory Bow
‘volume curves. There is no immediate response to bronchodilator. Du.co is mildly reduced, consistent with a pulmonary parenchymal or vascular process.
cna عد جاه وه ووه امه سا و
صفحه 49:
Restriction due to Restriction due to. Restriction due to Obstruction Obstruction due to
increased lung Chest wall respiratory muscle due to airway decreased
clastic recoil ‘abnormality weakness narrowing elastic recoil
(pulmonary (moderate (myasthenia (acute (severe
fibrosis) obesity) gravis) asthma) emphysema)
Te 60% و 73 100% 130%
FRC 60% 65% 100% 104% 220%
RV 60% 100% 120% 120% 810%
Fve 60% 92% 60% 60%
95% pre-b.d.
FEV, 750 92% 60% expla
Pow 1.0 1.0 1.0 15
ما 60% 9596 20 120% 40%
۳ 3
۱ x ‘|
FIGURE 306e6 Common abnormalities of | ا function (see اس Pulmonary function values are expressed as a percentage of
normal predicted values, except for R._, which is expressed as cmH,O/L per sec (normal, <2 cmH,O/L per second). The figures at the bottom
of each column show the typical configuration of flow-volume loops in each condition, including the flow-volume relationship during tidal
breathing, b.d, bronchodilator; يه diffusion capacity of lung for carbon monoxide; FEV, forced expiratory volume in 1 sec; FRC, functional
residual capacity; FVC, forced vital capacity; R, airways resistance; RV, residual volume; TLC, total lung capacity.
al.
é
__fiow
صفحه 50:
7 ۸ 8 6 D E
5 6
3
5 و
لا 2
2 1
2 و
Set
22
£3
a
2 4
£5
6
tf // ۳ ts .5. 7
100 0100 0100 0100 0100 0
Vital capacity (%)
Schematic flow-volume loop configurations in a spectrum of airway
lesions. A is normal; 8 is variable extrathoracic upper airway obstruction; C is variable
intrathoracic upper airway lesion; D is fixed upper airway obstruction; and E is small
airway obstruction. L/S = liters per second; = ventilation.
صفحه 51:
Nephrology: OSCE &
PMP
صفحه 52:
آقای ۵۷ ساله با شکایت تیره شدن رنگ ادرار و ادم پیش رونده اندام تحتانی. در معاینه اولیه فشار خون بالا داشته و
معاینات قلب وپوست وشکم طبیعی است. سمع ریه رال منتشر دارد. بررسی های اولیه به شرح زیر است:
aM ۷4 ی ات ی ی
5
111 VF. 13 ۷6
RHYTHM STRIP: 11
25 ۳۳/۵۵۵: ۱
صفحه 53:
نوار را تفسیر نموده و ۳ يافته مهم آن را نام ببريد:
MAT:
The cardinal features are irregularity and a plethora of different P-
wave morphologies.
You need to have
*Tachycardia (HR >100)
*Irregular rate
“Variability in P wave morphology and PR length
صفحه 54:
هماچوری +
پروتینوری +
گلوکوزاوی+
ييوردى
3
8
8
Urine Dipsteak:
“| DPE 0000 7
60 seclseg "029 1.005 1020,
7 Bo SH
co se
00 50
ee 5-06 a 5
و =
0 6275 ca. &
Biood/Hemoglobin/
eo 11111
ca. 25 50 80
Nitritenitritonitrtos ۴
0
Ketones!
C.Cetinicos
60 seciseg,
Bilirubin/Bilirrubina/
60 seclseg.
Ubilinogen(oy ">
Utobitinogénio
160 seciseg.
Protein/Proteinas/
Prooinas
00 seen 15 (0.15) 30/0. 100 300) 1000
نا )30:03 )015( سس تسه 0 1000/1) molt (ot)
Gicose
60 sex/seg
‘normal 100(&5) 300(17) 1000 55) mg/dl (nmol)
صفحه 55:
Urinary Sediment:
صفحه 56:
صفحه 57:
Diffuse pulmonary
haemorrhage with
Bilateral GGO
صفحه 58:
صفحه 59:
صفحه 60:
چه درمانی در این مرحله برای slay پیشنهاد میکنید؟؟
target. Between 10 and 15% of sera from patients with Goodpasture’s
syndrome also contain ANCA antibodies against myeloperoxidase.
This subset of patients has a vasculitis-associated variant, which has a
surprisingly good prognosis with treatment. Prognosis at presentation
is worse if there are >50% crescents on renal biopsy with advanced
fibrosis, if serum creatinine is >5-6 mg/dL, if oliguria is present, or if
there is a need for acute dialysis. Although frequently attempted, most
of these latter patients will not respond to plasmapheresis and steroids.
Patients with advanced renal failure who present with hemoptysis
should still be treated for their lung hemorrhage, as it responds to
plasmapheresis and can be lifesaving. Treated patients with less severe
disease typically respond to 8-10 treatments of plasmapheresis accom-
panied by oral prednisone and cyclophosphamide in the first 2 weeks.
Kidney transplantation is possible, but because there is risk of recur-
rence, experience suggests that patients should wait for 6 months and
until serum antibodies are undetectable.
صفحه 61:
Alz2y.o girl with sudden onset of generalized edema and
صفحه 62:
پسر ۲۳ ساله با نارسایی پیشرونده کلیه وهماچوری
و شرح حال مثبت خانوادگی از نظر 2580
صفحه 63:
B
پسر ۱۵ ساله با هماچوری میکروسکوپی پایدار و پروتینوری خفیف
هر کی بگه جایزه داره!
صفحه 64:
: ساله با چندین نوبت بررسى جهت هماچوری میکروسکوپی 52 ila
1.توصیف دقیق رادیولوژی
صفحه 65:
آقای 54 ساله 255 ESRD تحت
دیالیز با سابقه بستری 2 ماه قبل به علت
سر درد های راجعه شدید در یک
بیمارستان تخصصی اعصاب که از 2
هفته قبل دچار ضایعات پیشرونده پوستی
شده.
1.تشخیص
2.علت
3. توصیه های پیشگیرانه احتمالی
A rate complication, nephrogenic systemic fibrosis (NSE), has
recently been reported in patients with renal insuffi
exposed to gadolinium contrast agents, The onset of NSF has
and 75 days following exposure; histologic
features include thickened collagen bundles with surrounding
clefts, mucin deposition, and increased numbers of fibrocytes and
clastic fibers in skin. In addition to dermatologic symptoms, other
manifestations include widespread fibrosis of the skeletal muscle,
bone, lungs, pleura, pericardium, myocardium, kidney, muscle,
bone, testes, and dura. For this reason, the American College of
Radiology recommends that prior to elective gadolinium-based
MR contrast agent (GBMCA) administration, a recent (eg. past 6
weeks) glomerular filtration rate (GFR) assessment be obtained in
patients with a history of
iency who have
1. Renal disease (inchuding solitary kidney, renal transplant, renal
tumor)
2. Age 260 years
3. History of hypertension
4. History of diabetes
5. History of severe hepatic disease/liver transplant/pending liver
transplant: for these patients itis recommended that the patient's
GFR assessment be nearly contemporaneous with the MR
The incidence of NSF in patients with severe renal dysfunc
tion (GFR <30) varies from 0.19 to 4%. A recent meta-analysis
reported an odds ratio of 26.7 (95% Cl = 10,3-69.4) for develop-
ment of NSF after gadolinium administration in patients with
impaired renal function (GFR <30 mL/min/1.72 m). Thus, it
is not recommended to administer gadolinium to any patient
with a GRF below 30. Caution is advised for patients with a GRF
below 45,
صفحه 66:
آقای 72 ساله با سابقه سر درد های راجعه با شکایت تورم پیشرونده اندام تحتانی مراجعه نموده است. در
آزمایشات اولیه برای وی کرانینین ۵.۳ کشف شده است.
72-year-old male with
progressive analgesic
nephropathy. SICK (small,
indented, and calcified
kidneys) criteria was
present. Image shows
increased parenchymal
thickness, coarse
papillary calcifications
bilaterally, and numerous
indentations.
صفحه 67:
کدام یک از علل زیر در پاتوژنز نمای
میکروسکوپیک مفابل در آقای ۴۳ ساله
که با هماچوری و پروتینوری مراجعه
نموده است نقش اساسی دارد؟؟
63 ۲60۳۲9۳
Antiphospholipase a3
Ab
Ig A deposits
Hypocomplementemia
Light & electron microscopic
patterns of dense deposit disease
صفحه 68:
آقای ۲ ساله با درد فلانک چپ
Renal Tuberculosis
left autonephrectomy
secondary
to renal tuberculosis and
~@ probable cavitary
tuberculosis of the left
upper lobe was made. In
countries in which
index of suspicion for
tuberculosis in patients with
صفحه 69:
38-year-old patient presented to the
hospital with a 2-month history of fever,
malaise,weight loss, and progressive
_ weakness
Bilateral Foot Drop in Polyarteritis Nodosa
صفحه 70:
5
و
a
4
J
3
هاى مكرر ادرار
صفحه 71:
آقای ۵۴ ساله با سابقه بای پس کرونر ۷ روز قبل و درید اندام
صفحه 72:
A 37-year-old man was referred.
Laboratory evaluation revealed a
serum potassium level of 3.3
mmol per liter, a bicarbonate
level of 16 mmol per liter, a
calcium level of 9.3 mg per
deciliter (2.3 mmol per liter), a
phosphate level of 2.1 mg per
deciliter (0.7 mmol per liter), a
creatinine level of 3.0 mg per
deciliter (265 mol per liter), a
parathyroid hormone level of 62
Pg per milliliter, and an estimated
lomerular filtration rate of 25 ml 1
6. دأده م أكءاقع0 تامعط لطاع انها مأ ,5أكه ل زمی وهای دیا ۲625 دوع اق طز 5
gni€aeatabed is limited to the renal medulla. Three years after
sodium bicarbonate and potassium supplementation was
restarted, the patient's renal function has remained stable.
صفحه 73:
A 44 y,o man with positive familial
history for ESRD
صفحه 74:
Infectious Disease
صفحه 75:
Gram Negative Cell Wall
a لاقت ممعم
و
2 17
۰ ۵
Sep
Coban iad
مد ملسو
«سس ل
Sep
evil
‘pis opie
Sete be
اس سس
سس
ایور
و
سد سدم
سس
سس
wa
‘Sep
Talat
ste te wate
=
Gram Positive Cell Wall,
Sep,
ae
3
اس مروت
د حم
۳
pune مس روا
Sepa
Sat vee
‘aka compl +
meine
سس دس
تست
موه
Toys
olin comgtxis
ی نمی
لب هه تفر
نت
Sent
ال مت رعاو
عمج لس
ی یز
Gram Staining Procedure
سید | سای سس
cae
Sep lz
Begin witha | (
feed cells
Step2:
Flood slide
swith crystal &
ار مت
Seep 5
Counter eta
که شا
صفحه 76:
a. Fever, lymphadenopathy, meningismus
b. Carditis
c. Ischemic bowel
d. Cranial nerve palsies
e. Arthritis
The lesions in this patient developed 2 weeks after hiking in
the woods of the Upper Midwest in April. If left untreated, this
patient is at risk for development of all of the following except:
صفحه 77:
Lyme Disease
Answer: c
* Causative agent, Borrelia burgdorferi, is transmitted by Ixodes
ticks
‘+ Highest incidence is during spring and summer
* Tick must be attached more than 24 hours for transmission
to occur
+ Stage 1: Within 30 days of infection, erythema chronicum
migrans occurs in 80% of patients and may be associated
with fever, lymphadenopathy, and meningismus. The rash
resolves within 4 weeks
* Stage 2: Onset is weeks to months after stage 1. Neurologic
abnormalities occur in 10% to 15% of patients. Carditis occurs
in 10% of patients
* Stage 3: Begins months to years after infection. Arthritis,
‘occurs in 50% of untreated patients, becoming chronic in
10% to 20%
+ Enzyme-linked immunosorbent assay is positive for diag-
nosis within 2 to 6 weeks
* Early-stage disease may be treated with doxycycline, amox-
| oF cefuroxime
+ Neurologic or cardiac involvement requires high-dose cef-
triaxone or penicillin G
صفحه 78:
a. Fever
| b. Positive blood cultures
| Hypotension
d. Involvement of 3 or more organ syster
e. Erythroderma
Clinical criteria for toxic shock syndrome include all of the
following except:
صفحه 79:
Toxic Shock Syndrome
Answer: b
+ Acute life-threatening illness caused by production of
staphylococcal exotoxin (toxic shock syndrome toxin-1,
TSS)
+ Centers for Disease Control and Prevention criteria:
6 required for confirmation; 5 of 6 indicate a probable case
1. Fever ‘Temperature of more than 38:9°C
2 Rash Diffuse, macular, and erythematous
3. Desquamation Especially of palms and soles
4. Hypotension In adults, systolic blood pressure
less than 90 mm Hg
5. Multisystem Severe myalgias
involvement Diarthea, vomiting,
Liver dysfunction
‘Thrombocytopenia
Renal insufficiency
Mental status changes
6. Exclusion of other Negative results of blood, throat,
causes cerebrospinal fluid cultures
{usually not positive for
Staphylococcus aureus)
‘+ Associations: menstruation with prolonged use of tampons,
surgery (even if wound is not worrisome), bartier contracep-
tives
‘+ With early and aggressive management and judicious use of
antibiotics and supportive care, the mortality rate for toxic
shock syndrome has been estimated at 3%. ‘The use of
corticosteroids (to lessen the overwhelming inflammatory
]مرس رون anid Yaamiricgobuilay eeniaine م0050
صفحه 80:
خانم 30 ساله با شکایت تب و سر درد وترشحات
Soe فراوان از بینی از 15 روز قبل که به درمان
حمایتی جواب نداده است :
توصیف و تشخیص؟
درمان پیشنهادی؟
صفحه 81:
GUIDELINES FOR THE DIAGNOSIS AND TREATMENT OF ACUTE
‘SINUSITIS IN ADULTS:
Griteria__Treatment Recommendations*
Moderate symptoms Initial therapy:
eae Amoxicillin, 500 mg PO tid; or
ب Amoxkillin/clavulanate, $00/125 mg PO tid or
Severe srmptomsofary 875/125 mg PO bie
duration, including uni- Penicillin allergy:
lateral/focal facial swell- Doxycycline, 100 mg PO bid; or
ing oF tooth pain ‘Clindamycin, 300 mg PO tid
Exposure to antibiotics within 30 d or >30%
prevalence of penicilin-resistant Streptococcus
pneumoniae:
‘Amoxicillin/clavulanate (extended release),
200/125 mg PO bid; or
‘An antipneumococcal fluoroquinolone
(eg, moxifloxacin, 400 mg PO daily)
Recent treatment failure:
‘Amoxicillin/clavulanate (extended release),
2000 mg PO bid; or
‘An antipneumococcal fluoroquinolone
(eg, moxifloxacin, 400 mg PO daily)
“The duration of therapy is generally 7-10 days (with consideration ofa S-day course),
with appropriate follow-up. Severe disease may warrant IV antibiotics and consideration of
hospital admission. *Although the evidence is not as song, amoxiilin/clvulanate may
be considered fr inal use, particularly focal rates of peniilin resistance or (lactamase
production are high.
صفحه 82:
آقای 43 ساله مورد لنفوم غير
هوچکین که 5 ماه قبل آخرین سیکل
کموتراپی وی به اتمام رسیده است
از 2 ماه قبل دچار ضایعات پوستی
زیر و درد پیشرونده مچ پای راست
شده است. نمونه گیری انجام شده را
در زیر مشاهده میکنید.
درمان پیشنهادی؟
صفحه 83:
9-4 Nocardia brasiliensis mycetoma. A. Draining sinuses and giant white grains with a seropurulent discharge. B. Radiography of
the foot showing marked soft tissue enlargement and bony iytic lesions. € Direct microscopy of grains stained with Lugol's iodine (40)
D. Periodic acid-Schiff stain of skin biopsy (X40). (Image provided by Roberto Arenas and Mahreen Ameen, St. John's Institute of Dermatology, Gi
ys&
صفحه 84:
FIGURE 199-3 Nocardial abscesses in the right occipital lob:
FIGURE 1991 Nocardial pneumonia. A dense infiltrate w!
ible cavity and several nodules are
صفحه 85:
Nocardial pneumonia.
ws bilateral nodules, wit
صفحه 86:
TREATMENT DURATION FOR NOCARDIOSIS
Disease Duration
Pulmonary or systemic
Intact host defenses 6-12 months
Deficient host defenses 12months*
CNS disease 12 months!
Cellulitis, lymphocutaneous 2 months
syndrome
Osteomyelitis, arthritis laryngitis, 4 months
sinusitis
Actinomycetoma 6-12 months after clinical cure
Keratitis Topical: until apparent cure
‘Systemic: until 2-4 months after
apparent cure
‘tn some patients with AIDS and CD44. lymphocyte counts of <200/uiL oF with chronic
granulomatous disease, therapy for pulmonary or systemic disease must be continued
Indefinitely. fall apparent CNS disease has been excised, the duration of therapy may be
reduced to 6 months
At the outset, 10-20 mg/kg of
TMP and 50-100 mg/kg of SMX are given each day in two divided
doses. Later, daily doses can be decreased to as little as 5 mg/kg
and 25 mg/kg, respectively. In persons with sulfonamide allergies,
desensitization usually allows continuation of therapy with these
effective and inexpensive drugs.
صفحه 87:
a. Rifabutin
b. Trimethoprim-sulfamethoxazole
c. Pyrimethamine
d. Fluconazole
e. Acyclovir
A37-year-old man with acquired immunodeficiency syndrome
(AIDS) presents with fever, seizures, and altered mental status.
His last known CD4 cell count was less than 50 cells/mm3.
Which one of the following is the best prophylactic agent for
this condition?
صفحه 88:
Central Nervous System Toxoplasmosis
Answer: b
* The preferred prophylactic agent for Pneumocystis carinii and
Toxoplasma gondii is trimethoprim-sulfamethoxazole
* If the patient is allergic to sulfonamides, pyrimethamine and
dapsone are efficacious
* Without prophylaxis, central nervous system toxoplasmosis
will develop in 30% to 50% of patients with AIDS who are
seropositive for Toxoplasma IgG
* Magnetic resonance imaging is more sensitive than computed
tomography for detecting central nervous system lesions of,
toxoplasmosis
* Typical appearance is of multiple ring-enhancing lesions
within the brain parenchyma
* Central nervous system lymphoma can mimic toxoplasma
encephalitis,
* Patients with a single lesion, who are seronegative, or who
do not respond to antitoxoplasmosis therapy should be
referred for brain biopsy
* On biopsy, immunoperoxidase stain shows cysts and tachy-
zoites of T. gondii
صفحه 89:
. Imipenem
. Oxacillin and gentamicin
Vancomycin and gentamicin
|. Ceftriaxone and doxycycline
. Amphotericin B
]
°
In this case, in addition to diagnostic testing, which empiric
therapy is most appropriate?
صفحه 90:
Sexually Transmitted Urethritis
Answer: d
* Urethral discharge with burning on urination and an itchy
urethra are typical symptoms
Neisseria gonorrhoeae: Gram-negative intracellular
diplococcus
Diagnosis by molecular gene probe,
Gram stain, or culture
Injectable ceftriaxone is first-line
therapy
Add doxycycline or azithromycin for
treatment of Chlamydia trachomatis
because 15% to 25% of patients with
gonorrhea also have Chlamydia
‘Nongonoccocal urethritis: Symptomatically indistinguishable
from gonococcal variety, but much
more frequent
Most common causes are
C. trachomatis (25%-40%) and
Ureaplasma urealyticura (40%-50%)
Other causes include Mycoplasma
-genitalium, Trichomonas vaginalis,
and herpes simplex virus,
+ Sexual partners also should be treated and intercourse
avoided until treatment is completed
صفحه 91:
1. What was the alternative medication?
Ja. Pentamidine
1b. Pyrimethamine-sulfonamide
Ie, Atoyaquone
1d. Dapsone
Je. Clindamycin-primaquine
2, In addition to supplemental oxygen, which one of the
following would be the most appropriate antidote?
2a. N-Acetyleysteine
2b. Amyl nitrite
2c. Methylene blue
2d. Deferoxamine
2e. 4-Methylpyrazole
A patient with human immunodeficiency virus had a rash in
association with trimethoprim-sulfamethoxazole used as pro-
phylaxis for Pneumocystis carinii pneumonia. An alternative
medication was used, but the patient took 3 times the normal
dose by mistake. He became dyspneic, and headache, nausea
and vomiting, and the skin discoloration shown here devel-
oped (a normal hand is shown for contrast on the right).
صفحه 92:
Methemoglobinemia Due to Dapsone Overdose
Answer: d
Answer 2:
‘+ Methemoglobin is the ferric form of hemoglobin (HbFe>+)
that cannot bind oxygen
+ Although sometimes inherited, it is more often formed by
oxidative stress
+ Among drugs, nitrites are commonly associated with this
condition
+ Chocolate-brown or slate-blue cyanosis unrelieved with
oxygen suggests the diagnosis
Arterial blood gas studies will show normal arterial oxygen
pressure but low oxygen saturation. Pulse oximetry results
‘ill be inappropriately increased (the pulse oximeter result
will be higher than the actual oxygen saturation, although
the absolute value may still be below normal)
* Methylene blue is the antidote for patients with severe
hypoxia, unless they have glucose-6-phosphate dehyciro-
genase deficiency, in which case it may worsen the clinical
condition. If the patient has this deficiency, ascorbic acid
can be used to reduce the methemoglobin
۰ N-Acetyleysteine is the antidote for acetaminophen overdose
‘+ Amyl nitrite is an antidote for cyanide overdose because
nitrites precipitate the formation of methemoglobin, which
isan alternative binder for cyanide, Thus, it would be con-
traindicated in methemoglobinemia
+ Deferoxamine is the antidote for iron overdose
+ 4-Mothylpyrazole is an alternative to ethanol for the treat-
ment of methanal toxicity
صفحه 93:
a. Conversion disorder
b. Left brain abscess
c. Seizure with Todd paralysis
d. Left middle cerebral artery occlusion
e. Atypical migraine
A31-year-old man with Streptococcus viridans endocarditis has
acute right hemiparesis and dysarthria. Which one of the fol-
lowing is the most likely cause?
صفحه 94:
Left Middle Cerebral Artery Occlusion Due to
Embolus in Infective Endocarditis
Answer: d
‘+ Extracardiac manifestations of infective endocarditis includ
the following:
Embolic events
Suppurative complications
Immunologic reactions
Osler’s nodes (which are painful)
Roth’s spots
Increased rheumatoid factor
Glomerulonephritis
‘+ Systemic embolization occurs in 20% to 40% of patients wil
left-sided endocarditis
+ Embolization may occur at any time, but risk decreases wit
duration of treatment
‘Specifically vascular phenomena associated with infective
endocarditis include the followin,
Arterial emboli
Mycotic aneurysms
Intracranial hemorrhages
Conjunctival hemorrhages
Pulmonary infarcts (septic)
Janeway lesions (hemorthagic lesions on the palms or
soles, usually painless)
Splinter hemorrhages
صفحه 95:
a. Magnetic resonance imaging of the head
b. Computed tomography with thin cuts of the adrenal glands
ی Small bowel biopsy with periodic acid-Schiff staining
d. Dexamethasone suppression test
e. Cosyntropin stimulation test
١
A 40-year-old man presents with hyperpigmentation, malab-
sorptive diarrhea, weight loss, recurrent arthritis, and adenopa-
thy. His wife is shown for contrast. Which one of the following
tests is likely to confirm the diagnosis?
صفحه 96:
Whipple's Disease
Answer: ¢
‘+ Whipple's disease is a chronic relapsing, systemic infectious
disease involving the central nervous system, heart, kidney:
and small bowel
» Tt occurs primarily in white middle-aged men
+ The causative agent is a gram-positive bacillus, Tropheryma
whippelit
+ The name of the bacillus was chosen from the Greek word
trophe, which means “nourishment,” thus highlighting the
primary clinical feature of malabsorption
‘+ From 40% to 50% of patients have hyperpigmentation in
‘sun-exposed areas and scars
+ Other signs and symptoms include weight loss (80%),
diarthea (75%), arthralgia (70%), lymphadenopathy (55%),
abdominal tenderness (50%), fever (40%), edema (25%),
Blossitis (20%), splenomegaly (10%), ascites (5%)
‘+ 90% of patients have steatorrhea on 72-hour stool studies
+ Anemia of chronic disease and thrombocytosis are common
‘Joint symptoms may precede intestinal manifestations by years
+ Diagnosis is established with small bowel biopsy showing
periodic acid-Schiff-positive granules in macrophages with
‘gram-positive acid-fast bacillus-negative bacilli
‘+ Treatment is with trimethoprim-sulfamethoxazole for 1 year
‘+ Response to antibiotics is dramatic; most symptoms com-
pletely resolve within 2 to 4 weeks
‘© Relapses are common (up to 40%)
صفحه 97:
. Blood cultures and empiric therapy with vancomycin,
gentamicin, and metronidazole
. Blood cultures, supportive care, lumbar puncture, and
empiric doxycycline
. Supportive care until serologic diagnosis can be made to
direct therapy
d. Discussion with family about withdrawal of support and
palliative care
. Transesophageal echocardiography
3
This 25-year-old man was hiking in North Carolina 1 week
before development of rash and prostrating illness, including
hypotension, meningoencephalitis, anemia, and bleeding.
What is the most appropriate course of action?
صفحه 98:
Rocky Mountain Spotted Fever (RMSF)
Answer: b
* Although first described in Idaho in 1896, RMSF is most
common in the mid-Atlantic states and Oklahoma, not the
Rocky Mountains
۰ Rickettsia rickettsii, a gram-negative intracellular bacterium,
is inoculated into humans by ticks (Dermacentor variabilis
and D. anderson, the dog and wood tick, respectively) after
at least 6 hours of feeding
۰ Most cases (>90%) occur between April and September
* Incubation time is 2 to 14 days (mean, 7 days)
‘* Rash begins on the extremities and moves centrally
۰ Major symptoms and signs: fever (88%-100%), headache
(79%-93%), myalgia (72%-92%), rash (74%-90%), known tick
bite (54%-66%), nausea and vomiting (55%-60%), classic
triad of fever, headache, and rash (45%-60%), classic tetrad
of fever, headache, rash, and history of tick bite (3%-18%)
* Serologic test result is not positive until 7 to 10 days after
exposure; therefore treatment should not be delayed for
diagnosis
‘+ Mortality rate is 20% to 25% in 8 to 15 days without appro-
priate treatment and 5% despite appropriate treatment
‘+ Treatment of choice is doxycycline. Other tetracyclines and
chloramphenicol are also effective
صفحه 99:
صفحه 100:
پسر 6 ساله با شکایت تب و خواب آلودگی و ضایعات پوستی زیر به اورژانس
مراجعه نموده است. به گفته مادرش 3 روز قبل دچار ضایعات وزیکولار محدود
در ناحيه صورت شده است که رفته رفته پیشرفت نموده است :
1 .توصیف ضایعات و تشخیص؟
2.عامل بیماری و نام توکسین مربوطه؟
3.درمان؟
صفحه 101:
1
۱6۸۷۲۶ 172۰4 ۰ Evidence of staphylococcal scalded-skin syn-
drome in a 6-year-old boy. Nikolsiy’s sign, with separation of the
superficial laver of the outer epidermal laver. is visible, Reprinted with
STAPHYLOCOCCAL SCALDED-SKIN SYNDROME SSSS primarily affects new-
borns/and'children! The illness may vary from a localized blister to
exfoliation of much/Of the’skin(Ssurface: The skin is usually fragile
and often tender, with thin-walled, fluid-filled bullae. Gentle pres-
sure results in rupture of the lesions, leaving denuded underlying
skin (Nikolsky’s'sign; Fig. 172-4). Theimucous membranes are'usually
spared. In more generalized infection, there are often constitutional
symptoms, including fever, lethargy, and irritability with poor feeding.
Significant amounts of fluid can be lost in more extensive cases. Illness
usually follows localized infection at one of a number of possible sites.
5555 كز much less common among adults but can follow infections
caused by exfoliative toxin=producing strains:
THERAPY FOR OTHER TOXIN-MEDIATED DISEASES
Therapy for staphylococcal food poisoning is entirely supportive.
For SSSS, antistaphylococcal therapy targets the primary site of
infection.
صفحه 102:
پسر 10 ساله که هفته قبل به علت تب و لرز و سرفه (گرافی زیر) در بخش اطفال بستری و با تشخیص پنومونی تحت
درمان قرار گرفته است. 1 هفته بعد دچار ضایعات پوستی زیر به همراه درگیری مخاط دهان و بینی و چشم شده است.
1.عامل اولیه ایجاد پنومونی؟
2.نام سندروم پوستی؟
صفحه 103:
Sevens Soesen oyndronns i child wth Mycoplatme امتابم میم
(From Baum 5G. Mycoplasma preumoniae and atypial preumona. in Mandell GL.
ETT Etreng DIAGNOSTICTESTS FOR RESPIRATORY MYCOPLASMA
‘PNEUMONIAE INFECTION
Test Sensitivity, % Specificity, %
Respiratory culture <60 100
Respiratory PCR 65-90 90-100
Serologic studies" 55-100 55-100
ANTIMICROBIAL AGENTS OF CHOICE FOR MYCOPLASMA -212عاقم1
INFECTIONS:
Drugis)
‘Azithromycin, danthromycin, erythromycin,
doxycycline, levofloxacin, moxifloxacin,
‘gemifionacin (nor ciprofloxacin or ofloxacin)
U.ureabyicum, U.parum
M hominis Doxycycline, clindamycin
M.genitalium متهم نمم
‘Antimicrobial resistance has been reported in mycoplasmas, s described inthe text
صفحه 104:
limited, appropriate antimicrobial therapy significantly shortens the
duration of clinical illness. Infection uncommonly results in. critical
illness and only rarely in death. In some patients, long-term recur-
rent wheezing or reactive airway disease may follow the resolution of
acute pneumonia. The significance of chronic infection, especially as it
relates to asthma, is an area of active investigation.
Skin eruptions described with M. pneumoniae infection include ery-
thematous (macular or maculopapular), vesicular, bullous, petechial, and
urticarial rashes. In some reports, 17% of patients with M. pneumoniae
pneumonia have had an exanthem. Erythema multiforme major
(Stevens-Johnson syndrome) is the most clinically significant skin
eruption associated with M. pneumoniae infection; it appears to occur
more commonly with M, preurnoniae than with other infectious agents.
‘A wide spectrum of neurologic manifestations has been reported
With M. pneumoniae infection. The most common are meningoen-
cephalitis, encephalitis, Guillain-Barré syndrome, and aseptic menin-
gitis. M. pneumoniae has been implicated as a likely etiologic agent in
5-7% of cases of encephalitis. Other neurologic manifestations may
include cranial neuropathy, acute psychosis, cerebellar ataxia, acute
demyelinating encephalomyelitis, cerebrovascular thromboembolic
‘events, and transverse myelitis.
Hematologic manifestations of M. pneumoniae infection include
hemolytic anemia, aplastic anemia, cold agglutinins, disseminated
intravascular coagulation, and hypercoagulopathy. When anemia does
‘occur, it generally develops in the second or third week of illness.
In addition, hepatitis, glomerulonephritis, pancreatitis, myocarditis,
pericarditis, rhabdomyolysis, and arthritis (septic and reactive) have
been convincingly ascribed to M. pneumoniae infection. Septic arthri-
tis has been described most commonly in hypogammaglobulinemic
patients.
MYCOPLASMA PNEUMONIAE
CLINICAL MANIFESTATIONS
Upper Respiratory Tract infections and Pneumonia Acute M. pneumoniae
infections generally manifest as pharyngitis, tracheobronchitis, reactive
airway disease/ wheezing, ora nonspecific upper respiratory syndrome.
Little evidence supports the commonly held belief that this organism
isan important cause of otitis media, with or without bullous myringi-
tis, Pneumonia develops in 3-13% of infected individuals; its onset is
usually gradual, occurring over several days, but may be more abrupt.
Although Mycoplasma pneumonia may begin with a sore throat, the
‘most common presenting symptom is cough. The cough is typically
nonproductive, but some patients produce sputum. Headache, mal-
aise, chills, and fever are noted in the majority of patients.
‘On physical examination, wheezes or rales are detected in ~80% of
patients with M. pneumoniae pneumonia. In many patients, however,
pneumonia can be diagnosed only by chest radiography. The most
common radiographic pattern is that of peribronchial pneumonia
with thickened bronchial markings, streaks of interstitial infiltration,
and areas of subsegmental atelectasis. Segmental or lobar consolida-
tion is not uncommon. While clinically evident pleural effusions are
infrequent, lateral decubitus views reveal that up to 20% of patients
have pleural effusions.
‘Overall, the clinical presentation of pneumonia in’ anv individual
Patient is not useful for differentiating M. pneumoniae pneumonia
from other types of community-acquired pneumonia, The possibility
of M. pneumoniae infection deserves particular consideration when
‘community-acquired pneumonia fails to respond to treatment with a
penicillin or a cephalosporin—antibiotics that are ineffective against
mycoplasmas. Symptoms usually resolve within 2-3 weeks after the
onset of illness. Although M. pneumoniae pneumonia is generally self-
صفحه 105:
آقای 4 ساله باغبان و دامدار که از 4 روز
قبل دچار تب خفیف و ضایعه دردناک و
خارش دار گسترش یابنده در انگشت 3و4
دست راست شده است که ابتدا حالت وزیکل
داشته و سپس حالت پلاک پیدا کرده است
صفحه 106:
‘The gram-positive aerobic rod Erysipelothrix rhusiopathiae is most
often associated with fish and domestic swine and causes cellulitis pri-
marily in bone renderers and fishmongers. E. rhusiopathiae remains
susceptible to most f-lactam antibiotics (including penicillin), eryth-
romycin, clindamycin, tetracycline, and cephalosporins but is resistant
to sulfonamides, chloramphenicol, and vancomycin. Its resistance to
vancomycin, which is unusual among gram-positive bacteria, is of
potential clinical significance since this agent is sometimes used in
empirical therapy for skin infection. Fish food containing the water
flea Daphnia is sometimes contaminated with M. marinum, which can
cause cellulitis or granulomas on skin surfaces exposed to the water
in aquariums or injured in swimming pools. Rifampin plus etham-
butol has been an effective therapeutic combination in some cases,
although no comprehensive studies have been undertaken. In addi-
tion, some strains of M. marinum are susceptible to tetracycline or to
trimethoprim-sulfamethoxazole.
00۳۳2۳ erysipeiod with ts characteristic purple, nonpurulent swelling of
‘the finger. Also known as whale finger or pork finger, this form of cellulitis caused by
Erysipelothrix rhusiopathiae should not be confused with streptococcal or staphylococcal
-erysipelas (see Fig. 441-4). (From Farrar WE, Wood MU, Innes JUA, Tubbs H. Infectious
Diseases: Text ond Color Ata. 2nd ed. New York: Gower Medical Publishing: 1992)
صفحه 107:
صفحه 108:
Endocrinology : OSCE ۳
صفحه 109:
A 41-year-old woman presents with fatigue, anxiety, palpitations, and a tremor in her hands. She has lost approximately
9 وها in the past 4 months and reports feeling more comfortable in cool environments. Her menstrual cycle has become
irregular, and her eyes are often irritated. The patient recalls that one of her grandmothers was briefly treated for a
thyroid problem years ago.
On physical examination, her pulse is 110 beats per minute and she is diaphoretic. She has eye irritation, with red
conjunctivae, and has lid lag and proptosis in both eyes. She also has the appearance of staring and the reflexes
characteristic of patients with hyperthyroidism. The remainder of the examination is normal. Laboratory testing reveals
hyperthyroidism; the patient has a low level of thyrotropin and an elevated level of free thyroxine. A thyroid scan is
ordered after the administration of radioactive (jodine-123)
Which one of the following diagnoses is most likely in this patient?
O Drug-induced thyroiditis
O Graves’ disease Y¥ O Graves’ disease
© Painful subacute thyroiditis
O Toxic adenoma
O Toxic multinodular goiter
Tracer uptake was 65% within 6 hours after
the administration of iodine-123. The scan
revealed a homogeneous pattern of uptake.
صفحه 110:
The correct answer is Graves’ disease. Close [x]
Graves’ disease is an autoimmune disorder caused by antibodies that bind to and
activate G-protein—coupled thyrotropin receptors, stimulating the development of
hypertrophy and hyperplasia in thyroid follicular cells. This stimulation causes both
enlargement of the thyroid and an increase in the production of thyroid hormones.
Graves’ disease is the most common cause of moderate-to-severe hyperthyroidism and
is seen predominantly in women between the ages of 40 years and 60 years of age.
Graves’ ophthalmopathy is clinically apparent in approximately 50% of patients with
Graves’ disease. Patients have symptoms of hyperthyroidism, including weight loss, heat
intolerance, difficulty sleeping, tremor, diarrhea, irritability and anxiety, and menstrual
irregularity. Physical signs include tachycardia, goiter, resting tremor, hyperreflexia, and
warm, moist, and smooth skin. In the eyes, signs include lid lag, proptosis, periorbital
edema, conjunctivitis, erythema, and retraction of the upper eyelid. In rare instances,
pretibial myxedema can also be detected.
صفحه 111:
A 25-year-old woman with no history of thyroid disease presents with a 5-day history of pain in her neck. She reports the
development of a viral infection of the upper respiratory tract 1 week before presentation, with tendemess to palpation in
her neck developing a few days later. She also reports having intermittent palpitations.
On physical examination, she is febrile, with temperatures as high as 38.9°C, and her pulse is 105 beats per minute. She
is also diaphoretic. There is no lid lag or appearance of staring. The thyroid gland is diffusely tender to palpation, and
there is a slight resting tremor in her hands. Reflexes are brisk. The remainder of the examination is normal. Laboratory
testing reveals hyperthyroidism; the patient has a low level of thyrotropin and an elevated level of free thyroxine. A thyroid
scan is ordered after the administration of iodine-123.
Which one of the following diagnoses is most likely in this patient?
© Drug-induced thyroiditis
O Graves’ disease
O Painful subacute thyroiditis“ () Painful subacute thyroiditis
O Toxic adenoma
O Toxic multinodular goiter
View cor
No measurable tracer uptake had occurred 6
‘hours after the administration iodine-123.
صفحه 112:
The correct answer is painful subacute thyroiditis. Close
Painful subacute thyroiditis, also known as de Quervain's thyroiditis, is often but not
necessarily preceded by viral infection of the upper respiratory tract. Typically, thyroiditis
has three phases. The first phase is characterized by hyperthyroidism, which results
from follicle destruction and the release of preformed thyroid hormone into the
circulation. This phase often occurs before the clinical presentation. The second phase
is اه ای hypothyroidism, which results from the loss of thyroid-hormone
stores. In the third pha: re is resolution, with the return of euthyroidism (typically in
3 to 5 months). However, permanent hypothyroidism develops in approximately 5% of
affected patients.
Subacute thyroiditis is generally self-limited and is not treated with radioiodine or
antithyroid drugs. Treatment addresses symptoms and is administered as needed,
usualy in the form of a beta-adrenergic-receptor blocker.
صفحه 113:
‘A38-year-old woman with long-standing asymmetric enlargement of her neck (goiter) reports palpitations, a gradually
increasing tremor in her hands, heat intolerance, and a loss of 4.5 kg over the past 6 months. She has no history of
irradiation of the head and neck. Her grandmother also had a goiter.
(On physical examination, her pulse is 108 beats per minute and her skin is moist and warm. She has mild lid lag but no
appearance of staring. The mobile thyroid nodule on the left side of her neck measures, 5 cm in its largest diameter; there
is no lymphadenopathy or bruit, and the nodule is nontender to palpation. Her hands show a slight tremor when at rest.
Her reflexes are brisk, and the remainder of the examination is normal. A thyroid scan is ordered after the administration of
jodine-123. The results of laboratory tests reveal hyperthyroidism.
Which one of the following diagnoses is most likely in this patient?
O Drug-induced thyroiditis,
O Graves’ disease
O Painful subacute thyroiditis
225
3 O Toxic adenoma CE Toxic adenoma
57 O Toxic multinodular goiter
a كفن | Adele act indre-28 wis
Toxic adenoma is a functional benign tumor of the thyroid that produces excessive Tracer uptake was 40% within 6 hours after
amounts of thyroid hormone. the administration of iodine-123,
صفحه 114:
within 6 hours after the administration of
‘50-year-old woman who has had an asymmetric, enlarged goiter for many years reports the development of
intermittent palpitations with an irregular heartbeat over the past 6 months. She has also lost 4.5 kg during this period,
despite having a good appetite.
On physical examination, her skin is warm and moist. She has three palpable, mobile thyroid nodules that measure
between 2 cm and 4 cm in the largest diameter. There is no lymphadenopathy or bruit, and the thyroid is nontender to
palpation. Her heart rate is elevated at 144 beats per minute; the rate is imegularly irregular, and a systolic murmur can
be heard at the left lower sternal border. She has mild edema in both legs. Her reflexes are brisk, and the remainder of
the examination is normal. Laboratory testing reveals hyperthyroidism; the patient has a low level of thyrotropin and an
elevated level of free thyroxine. A thyroid scan is ordered after the administration of iodine-123.
Which one of the following diagnoses is most likely in this patient?
O Drug-induced thyroiditis
O Graves’ disease
© Painful subacute thyroiditis
O Toxic adenoma
O Toxic multinodular goiter _¥“ @) Toxic multinodular goiter
— 1 3 ‘Several focal areas of increased radioiodine
The correct answer is toxic multinodular goiter. ددن
۳7 0 uptake can be seen, with suppression of
The prevalence of toxic multinodular goiter increases with age and is endemic in surrounding tissue. Tracer uptake was 28%
iodine-123.
regions with ahistory of iodine deficiency, such as Eastem Europe. Toxic multinodular
adenomas may be treated with antithyroid drugs, radioiodine therapy, or thyroid
surgery.
صفحه 115:
A 54-year-old woman presents with increased anxiety, diarrhea, sweating, and hand tremors that began 3 months
earlier. She also lost 6.8 kg during this period. She has a history of atrial fibrillation and has been treated with
‘amiodarone for the past 2 years. She has no history of thyroid disease — thyroid function tests were normal before she
began treatment with amiodarone.
On physical examination, her skin is warm and moist. Her thyroid is normal and nontender and is without nodules. Her
hands show a slight resting tremor. Her heart rate is 80 beats per minute and regular, and there is a systolic murmur at
the left lower sternal border. She has mild edema in both legs. Her reflexes are brisk, and the remainder of the
‘examination is normal. Laboratory testing reveals hyperthyroidism; the patient has a low level of thyrotropin and an
elevated level of free thyroxine. A thyroid scan is ordered after the administration of iodine-123.
Which one of the following diagnoses is most likely in this patient?
O Drug-induced thyroiditis كي © Drug-induced thyroiditis
O Graves’ disease
O Painful subacute thyroiditis
O Toxic adenoma
O Toxic multinodular goiter
View commer
Tracer uptake was low (<5%) within 6 hours
after the administration of iodine-123.
صفحه 116:
The correct answer is drug-induced thyroiditis. Close
Amiodarone-induced thyrotoxicosis occurs in up to 20% of patients receiving the
drug. The condition may develop at any time during treatment and for up to 1 year after
discontinuation of the drug.
There are two types of amiodarone-induced thyroiditis. In type 1 disease, iodine induces
an increase in thyroid-hormone synthesis and release. It is most common in patients with
ahistory of subclinical thyroid disease and multinodular goiter. In type 2 disease, a
destructive inflammatory process causes the release of preformed thyroid hormones and
leads to thyroiditis. It is often challenging to distinguish the two forms of amiodarone-
induced thyroiditis, since both are associated with low uptake of radioiodine. Some
patients may have both types. On color-flow Doppler ultrasonography performed by an
experienced ultrasonographer, type 1 disease reveals hypervascularity and type 2
disease reveals reduced blood flow. The primary treatment for type 1 disease is high
doses of antithyroid drugs. Type 2 disease is treated with glucocorticoids.
صفحه 117:
آقای ۲۴ ساله با ضعف ,ء بی حالی و کاهش وزن به اورژانس شهدای تجریش مراجعه
نموده است. سابقه هیچ گونه بیماری را نداشته و از ۲ هفته قبل دچار تهوع و استفراغ
های مکرر شده که در آندوسکوپی وی به جز چندین کانون تیره رنگ در مخاط مری
نکته ای نداشته است, در شرح حال بیمار کدام یک از یافته های زیر محتمل ترند؟؟
8 درد مفاصل و میلژی ٠ پر نوشی و پرادراری
6 بی اشتهایی 6 تغییر رنگ پوست
٠ خوئريزى بينى ٠ كاهش تمايل به نمى
© هييوتانسيون وضعيتى
٠ مصرف سيكار
صفحه 118:
کدام یک از یافته های آزمایشگاهی در اين بیمار محتمل است؟
+ >
هیپرناترمی
۵.هپرکلمی
ADH Gli.
پلاسما
۲5۲۷ سطح ase
©.لنفوسيتوز
©.ايوزينوفيلى
.آلكالوز متابوليىك
صفحه 119:
کدام یک از, تست های زیر در تایید تشخیص در این مرحله اولویت بیشتری دارد؟؟
۱.الکترولیت های سرم و ©86© چم ۸.سطح آلدوسترون سرم .يه
۲.فعالیت رنین پلاسما 4و ٩.سطح سرمی اسید های چرب با زنجیره متوسط
۳.کورتیزول ادرار ۲۴ ساعته ۰ تست های عملکرد کلبه يم
chu! آدرنوکورتیکرتروپین سرم #ي» ۱ تست اينفيوزن سالين
A 2. TSH
۶.تست کوزینتروپین ۳
۷.تست سرکوب دگزامتازون با ۱ میلیگرم
صفحه 120:
رم أزمايشكاه موكزى باتوبيولوزى ۷ |
ا"
sige = 3
tay )2 سید مجتی نگوقدممطق
۳۳
صفحه 121:
]۳7۳[ ۶۱6۱/5 ۸۱۸۵ 5۷۱۸۵۲۵۱۷۸5 ۵۴ ۸۵۸6۱۷۸۸ ۱۸5۵۴۴۱۵۵۵۷6۲ ISURE#06-15 Clinical features of Addison's disease. Note the hyperpiamentation in areas of increased friction including (A) palmar creases
(8) dorsal foot, (C nipples and allay region, and () patchy hyperpigmentaten ofthe oral mucosa
Signs and Symptoms Caused by Glucocorticoid Deficiency
TB>Fatigue, lack of energy
Weight loss, anorexia
pain مامز ,جاورالا
Fever
Normochromic anemia, lymphocytosis, eosinophilia
Slightly increased TSH (due to loss of feedback inhibition of TSH release)
Hypoglycemia (more frequent in children)
Low blood pressure, postural hypotension
Hyponatremia (due to loss of feedback inhibition of AVP release)
‘Signs and Symptoms Caused by Mineralocorticoid Deficiency (Primary
‘Adrenal insufficiency Only)
“Abominal pain, nausea, vomiting
Diziness postural hypotension
Salt caving
Low blood pressure, postural hypotension
Increased serum creatinine (due to volume depletion)
Hyponatremia
Hyperkalemia
Signs and Symptoms Caused by Adrenal Androgen Deficiency
Lack of energy
Dry and itchy skin ¢n women)
Loss of ibido (in women)
Loss of allay and pubic hair (in women)
Other Signs and Symptoms
Hyperplamentation primary adrenal insufciency oni) (due to excess of
proopiomelanacortn [POMC]-derived peptides)
Alabaster-colored pale skin (secondary adrenal insufficiency only) (due to
deficency of POMC.denved peptides)
صفحه 122:
با توجه به اقدامات قبلی و نتایج به دست آمده در حال حاضر کدام تست تشخیصی را پیشنهاد میکنید:
Adrenal PET scan
Brain CT + Contrast
CXR Jo
Plasma VLCFA JS
17-OH progestron JS
Adrenal CT w/wo سای
Whole body bone scan
Check mutation of VHL
gene
صفحه 123:
مدالیته تصویر برداری- توصیف ضایعه- تشخیص
X-linked adrenoleukodystrophy has an incidence of 1:20,000
maleg and is caused by mutations in the X-ALD gene encoding the
feroxfsnrllhembrane transporter protein ABCD1; its disruption
results in accumulation of very long chain (>24 carbon atoms) fatty
acids. Approximately 50% of cases manifest in early childhood with
rapidly progressive white matter disease (cerebral ALD); 35% present
during adolescence or in early adulthood with neurologic features
indicative of myelin and peripheral nervous system involvement
(adrenomyeloneuropathy [AMN]). In the remaining 15%, adrenal
insufficiency is the sole manifestation of disease. Of note, distinct
mutations manifest with variable penetrance and phenotypes within
affected families.
صفحه 124:
کدام یک از درمان های زیر را در این مرحله پیشنهاد میکنید :
۱.تزریق هفتگی متیل پردنیزولون ۲۰ میلیگرم
۲.قرص هیدروکورتیزون ۱۵ میلیگرم صبح و ۱۰ میلیگرم شب JS
۳.امپول دگزامتازون ۸ میلیگرم ماهانه
۴.قرص پردنیزولون ۲۵ میلیگرم روزانه
۵.قرص فلودروکورتیزون ۱۰۰ میکروگرم روزانه 4
صفحه 125:
ا ا اك ب
1
۳
[0 agnosis
+ Pama corse 39-80 min stor 239 انه قل جاجد چیه ون
(Corso post رای 500 001
+ OBC, sarum cei, potactum, cearine, ura, TSH
‘ileal agnosia
Plana ACTH, ls عمد اه و
7[ ماد لمعيه سما
ow toro ACTH, ema i و
Store) tera! arene)
1 1
0
سجاوه ‘elcome cco
(wet rtany 0
نیمه reste I
Hse dt exponen
Senator?
Hypothalamic | | Elon of bod ture?
امعم وم | | Color lo ACTH
‘tency
صفحه 126:
A ۱ سا حملااةا سن ذرة: وتران قل وار :یاف هلى زير را توصيف نموده و تشخيص
احتمالى خود را بنويسيد :
The VHL gene (among other genes) encodes an E3 ubiquitin ligase
that regulates expression of hypoxia-inducible factor 1. Loss of VHL
is associated with increased expression of vascular endothelial growth
factor (VEGF), which induces angiogenesis. Although the VHL gene
can be inactivated by all types of mutations, patients with pheochro-
mocytoma predominantly have missense mutations. About 20-30% of
patients with VHL have pheochromocytomas, but in some families the
incidence can reach 90%.
صفحه 127:
Von Hippel-Lindau disease. A. Retinal angioma. All subsequent panels show findings on MRI B-D. Hemangioblastomas of,
the cerebellum (8) in brainstem (Q and spinal cord (D).E. Bilateral pheochromocytomas and bilateral renal clear cell carcinomas F, Multiple
pancreatic cysts. (Parts A and D from HPH Neumann et al: Adv Nephrol Necker Hosp 27:361, 1997, © Elsevier, Part B from SH Morgan, JP Grunfeld feds}:
صفحه 128:
صفحه 129:
آقای ۲۹ ساله با درد سینه ناگهانی» تعریق وتهوع از چند ساعت قبل به اورژانس
مراجعه نموده است. در بدو ورود نوار قلب زیر از بیمار اخذ شده است.
۱ ۱
صفحه 130:
سوالات حاشیه ای که ممکن است پرسیده شود:
توصیف دقيق و تشخيص؟؟؟
۲ اختلال الكتروليتى كه ممكن است الكوى نوارى مشابه ايجاد نمايند؟
كدام خانواده آنتى اريتميك قادر است باعث 57718 كردد؟
كدام سندروم ها باعث 5711 منحصرا در ليد هاى 1/1 تا3لا
ميكردد؟
[۲۱۲31۰3 DIFFERENTIAL DIAGNOSIS OF ST-SEGMENT ELEVATIONS
Ischemia/myocardial infarction
Noninfarction, transmural ischemia Prinzmetals angina, and probably
Tako-tsubo syndrome, which may alo exactly simulate classical acute
infarction)
Acute myocardial infarction
Postmyocardial infarction (ventricular aneurysm pattern)
Acute pericarditis
Normal variants (including benign ‘early repolarization’ pattems)
Left ventricular hypertrophy/left bundle branch block?
Other (aren)
Acute pulmonary embolism?
Brugada pattems (right bundle branch block-lke pattern with ST elevations
In right precordial leads)
Class 1C antiarthythic drugst
DC cardioversion
Hypercalcemia?
Hypetkalemiae
Hypothermia ( [Osborn] waves)
Nonischemic myocardial injury
Myocarditis
Tumor invading left ventricle
Trauma to ventricles
Usually localized لاه را
Source: Modified ror AL Goldberger etal Goluberger Clinical Electrocardioaraghy: A
‘Simplified Approach, 8 ed. Philadelphia lsevier/Saunders, 2013.
صفحه 131:
کدام یک از موارد زیر در اخذ شرح حال و تعيين پلان درمانی اين بیمار در اولویت است؟؟؟
0 0 0© 0 © © ©
سابقه فاميلى از نظر آترواسكلروز زودرس
مصرف سيكار
ابى زود هاى قبلى درد مینه مشابه
حملات مکرر درد شکم از کودکی
سابقه بيماريهاى روماتولوزيك در کودکی
مصرف تفريحى الكل
شدت درد
عفونت ویرال اخیر مجاری هوایی فوقانی
© سابقه مسرف كوكيين
CO) واكسيناسيون دوره كودكى
© سابقه خونريزى مغزى
قشار خون بالا
© سابقه سکته مغزی اخیر
© داروهاى مصرفى بيمار
©6 زمان دقيق شروع درد
صفحه 132:
در قدم بعد کدام یک از اقدامات زیر جهت بیمار ضرورت بیشتری دارد؟؟
0 9 0 00 © © © ©
شروع فیبریتولیتیک در صورت عدم کنتر اندیکاسیون
تست ورزش
نوار قلب سریال
چک آنزیم های قلبی
معاینه دقیق قلبی-عروقی-ریوی
ارسال نمونه ادرار
اسکن هسته ای قلب
پایدار ساختن راه هوایی
اکوکاردیوگرافی اورژانسی
()) ارسال آزمایشات اوليه و بروفايل جربى
© لس اكسيمترى
0 شروع ليدوكايين وريدى جهت جلوكيرى از أريتمى
() شروع أسبيرين و بلاويكس
© حفظ اشباع اكسيزن شريانى در حد ۹۸ درصد
OC سیتی آنژیوگرافی عروق کرونر
)) بررسی بیمار از نظر دیسکسیون أيورت
0 مانیتورینگ قلبی
(2) نتروگلیسرین زیرزبانی هر ۰ دقیقه تا ۲ ساعت
صفحه 133:
نتایج آزمایشات بیمار به شرح زیر میباشد :
LDL:349
HDL: 32
Cholest: 490
14
ELECTROLYTES: NI
CPK:232
MB:55
Tn: 12
ALT:29
AST:54
BILI:1.1
BUN: 21
Cr:1.1
ESR:12
CRP:11
CBC: NI
PT,PTT: NI
صفحه 134:
کدام یک از تشخیص های زیر جهت بیمار بیشتر مطرح است :
هیپرتریگلیسریدیمی فامیلیال
دیس بتالیپوپروتینمی
شیلومیکرونمی
سیتواسترولمی
هیپرلیپیدمی فامیلیال ترکیبی
صفحه 135:
: یک از یافته های زیر در معاینه بالینی اين بیمار مورد انتظار است plas
Oe
vA
صفحه 136:
Physical signs of
heterozygous familial
hypercholesterolemia,
as a result of cholesterol
deposition within
macrophages in specific
_ sites. Tendinous
xanthomas, for example,
manifests first as
thickening of, and later
as deposits within,
extensor tendons. A:
lateral borders of
thickened Achilles’
tendons are shown with
arrows. B: tendinous
xanthomas can also
occur in the extensor
tendons of the hands,
صفحه 137:
کدام یک از خانواده های دارویی زير در بیماران مبتلا به اين اختلال کاریردی ندارد؟
١ .استاتين ها 00 ٩ ازتیماب 00
۲فیرات ها as. So
5
۴.رژیم غذایی حاوى سو راي
00 LDL آفرز .4
FFP dust? هيه
۱3 وی inhibitors
Ginhibitirs of MTP
صفحه 138:
Patients with homozygous FH must be treated aggressively to
delay the onset and progression of CVD.(Receptor defective patients
sometimes respond to statins and other LDL-lowering drug classes
such as a cholesterol absorption inhibitor or a bile acid sequestrant,
which upregulate the LDL receptor activity. Two drugs that reduce the
hepatic production of VLDL and thus LDL,a small-molecule inhibitor
of the microsomal TG transfer protein (MTP) and an antisense oligo-
nucleotide to apoB, are approved in the United States for the treatment
of adults with homozygous FH and can be considered. PCSK9 inhibi-
tors, which work through increasing LDL receptor availability, appear
to have some benefit in receptor-defective patients and are under clini-
cal development. LDL apheresis is used to lower plasma LDL levels in
these patients and can promote regression of xanthomas as well as slow
the progression of atherosclerosis. Because the liver is quantitatively
the most important tissue for removing circulating LDLs via the LDL
receptor, liver transplantation is effective in decreasing plasma LDL-C
levels in this disorder but is infrequently used because of the associated
problems with immunosuppression.
صفحه 139:
آقای ۵۵ ساله دیابتی جهت بررسی تاری
دید پیشرونده مراجعه نموده است
توصیف ۳ يافته مهم- تشخیص-درمان
The most effective therapy for diabetic retinopathy is preven-
tion. Intensive glycemic and blood pressure control will delay the
development or slow the progression of retinopathy in individuals
With either type 1 or type 2 DM. Paradoxically, during the first 6~12
months of improved glycemic control, established diabetic retinop-
athy may transiently worsen. Fortunately, this progression is tempo-
rary, and in the long term, improved glycemic control is associated
With less diabetic retinopathy. Individuals with known retinopathy
may be candidates for prophylactic laser photocoagulation when
initiating intensive therapy. Once advanced retinopathy is present,
improved glycemic control imparts less benefit, although adequate
‘ophthalmologic care can prevent most blindness.
Regular, comprehensive eye examinations are essential for all
individuals with DM (see Table 418-1). Most diabetic eye disease
‘can be successfully treated if detected early. Routine, nondilated
eye examinations by the primary care provider or diabetes special-
ist are inadequate to detect diabetic eye disease, which requires an
ophthalmologist for optimal care of these disorders. Laser photo-
‘coagulation is very successful in preserving vision. Proliferative reti-
nopathy is usually treated with panretinal laser photocoagulation,
whereas macular edema is treated with focal laser photocoagula-
tion and anti-vascular endothelial growth factor therapy (ocular
injection). Aspirin therapy (650 mg/d) does not appear to influence
the natural history of diabetic retinopathy.
صفحه 140:
A 23-year-old woman was admitted with a 3-day history of fever, cough
productive of blood-tinged sputum, confusion, and orthostasis. Past medical
history included type 1 DM. Physical examination indicated postural
hypotension, tachycardia, and rapid respiration. Examination of the thorax
lobreptrectese] consolidatié¢vin the righPitwer lobe.
* توصیف کامل اختلال اسیدو باز بیمار
* محتملترین تشخیص
* تست درخواستی بعدی و درمان طبق
الگوریتم
should be
lic acidosis.
dilution of
with type 1
increase in
several
‘The diagnosis of DKA is usually not challenging but
considered in all patients with an elevated AG and metabol
Hyperglycemia and ketonemia (positive acetoacetate at a
1:8 or greater) are sufficient criteria for diagnosis in patients
diabetes mellitus. The A[HCO,”] should approximate the
the plasma AG (AAG), but this equality can be modified by
factors. For example, the AAG will often decrease with IV hydration, as
o the urine,
glomerular filtration increases and ketones are excreted
meq/L
meq/L
mea/L
امهم
mg/dL
9/4
ma/dL
mmHg
mmHg
امهم
Sodium 13
Potassium 50
Chloride 96
02 14
Blood urea nitrogen (BUN) 20
Creatinine 13
Glucose 450
Arterial Blood Gases On Room Air
pH 739
Poo, 24
Pao, 89
(HCO; 14
Urinalysis
Urine ketones Positive 4+
Glucose Positive 4+
Sarum Ketones Stronaly positive 1:8
صفحه 141:
۲۳۱۱۱۹۱۲۵۰ ۱۸۸۱۸۸۹۸۸۵۱۸۲ OF DIABETIC KETOACIDOSIS
1. Confirm hagrosis(T plasma glucose positive serum letones, metabalic
eciosi).
2. Admit ta hospital intensive care sting maybe necessary fr frequent
monitoring oF pH <7. 00 or uncanscous
ترچ
Serum slecilytes (Nat, Mg, Cr ieatbonat, phosphate)
یف status —pH, HCO, Peo, Prhyroxyburyrate
Renal function (creatinine urine ouput
4. Replace fis: 2-2 L of 096 saline over fist 1-3 h (10-20 ml/g per
hour), suosequently, 045% saline at 250-500 mL/h; change to 5% alu-
cose red 043% sane at 150-250 ml/h when plasma glucose reaches
250 mo/a. (139 TOI
5. Administer shortacting insulin 1V (01 units), then 0 unitsfig per
hour by continuous Vnfusion nctease wot tnecold if no resorse
by 2th the nel serum potas s-<33 mol (33 mec). do
not admin nsdn unt the possum ie camected
6. Assess patient What prectated the episode (noncomesiance, infection
trauma, pregnancy, infarction, cocare) inate aproprave workup fr
] evert (cultures, C3R, ECG)
7, Measure caoilry glucose ever 1-2 measure electrolytes (espectly
W bicabonate phosphate] and anion gap every 4h fest 28h
Monitor blood pressure, pulse resptatlons, ment status, id intake
and ouiputevery 1-4
9, Replace 6: 10 mea/h when plasma K* <5.0-52:meayl (or 20-30 mea/
of fusion Mui), ECG norma, une flow and normal cestinine ocr
mmenved: administer 40-20 mec/h vihen lesa K"<35 meey OF
Diearbonate fs gwen. inal eum potachmn 98.2 mcd.
(52 mer) do not supplement K" Unt the pass i corected
10. See ex about bicarbonate or phosphate supplementation
11, Continue abore until patient stable, lucose goals 22-139 mmol.
(050-250 mg/L), and acidosis resolved. sun infusion may be
decreased 16 095-01 unlsig per hout
12: Adiiniste long-acting insulin as scan as patient seating. Alow fra
oa hour overap i insulin infusion and SC insulin injection
صفحه 142:
‘A41-year-old woman presents with fatigue, anxéety, palpitations, and a tremor in her hands. She has lost approximately
‘9kg in the past 4 months and reports feeling more comfortable in coo! environments. Her menstrual cycle has become
irregular, ard her ayes are oftan irritated. The patiant recalls that one of her grandmothers was briafy treated for 2
thyroid problem years ago.
‘On physical examination, her pulse is 1/0 beets per minute and she is dicphoretic. She has eye initation, with red
cconjunctivae, and has lid lag and proptosis in both eyes. She also has the appearance of staring and the refexes
characteristic of patents with hyperhyroidism Tha remainder of the examination is normal. Laboratory testing reveals
hyperthyroidism; the patient has 2 low level of thyrotropin and an elevated level of free thyroxine. A thyroid scan is
ordered after the adminsstration of radioactive (iodine-123).
Which one of the following diagnoses is most likely in this patient?
O Drug-inducad thyroicitis,
CO Graves’ disease
© Painful subacute thyroiditis
(O Toxic adenoma
O Toxic multinodular goiter
Tracer uptake was 65% within 6 hours after
۰ the administration of iodine-123. The scan
V O Graves’ disease revealed a homogeneous patter of uptake.
صفحه 143:
A25-year-old woman with no history of thyroid disease presents with a 5-day history of pain in her neck. She reports the
development of a viral infection of the upper respiratory tract 1 week before presentation, with tendemess to palpation in
her neck developing a few days later. She also reports having intermittent palpitations.
‘On physical examination, she is febrile, with temperatures as high as 38.9°C, and her pulse is 105 beats per minute. She
is also diaphoretic. There is no lid lag or appearance of staring. The thyroid gland is diffusely tender to palpation, and
there is a slight resting tremor in her hands. Reflexes are brisk. The remainder of the examination is normal. Labor
testing reveals hyperthyroidism; the patient has a low level of thyrotropin and an elevated level of free thyroxine, A thyroid
‘scan is ordered after the administration of iodine-123.
Which one of the following diagnoses is most likely in this patient?
Drug-induced thyroiditis
CO Graves’ disease
© Painful subacute thyroiditis
O Toxic adenoma
O Toxic multinodular goiter
Y © Painful subacute thyroiditis | Romesnuratie aces tpl ae coon
صفحه 144:
۸ 3 0امعو ۱۱۵9۵0 ۷/۳ long-standing asymmetric enlargement of her neck (goiter) reports palpitations, a gradually
increasing tremor in her hands, heat intolerance, and a loss of 4.5 kg over the past 6 months. She has no history of
‘radiation of the head and neck. Her grandmother also had a goter.
On physical examination, her pulse is 108 beats per minute and her skin is moist and warm. She has mild lid lag but no
appearance of staring. The mobile thyroid nodule on the left side of her neck measures, 5 cm in its largest diameter; there
ig no lymphadenopathy or bruit, and the nodule is nontender to palpation. Her hands chow a slight ramor when at rest.
Hor reflexes aro brisk, and the remainder of the examination ie normal. A thyroid soan is orderad afior the administration of
odine-123. The results of laboratory tests reveal hyperthyroidism.
Which one af the follwing diagnoses is most likely in this patient?
O brugsinduced thyroiitis
O Graves’ disease
© Painful subacute thyroiditis
O Tox adenoma
© ‘oxic multinodular goiter
Adiscrete area of iodine-123 uptake is
۱ ‘noted, with no uptake in surrounding tissue.
Y O Toxic adenoma Tracer uptake was 40% within 6 hours after
the administration of iodine-123,
صفحه 145:
١ ‘A50-year-ald woman who has had an asymmetric, enlarged goiter for many years reports the development of
intermittent palpitations with an regular heartbeat over the past 6 months. She has also lost 4.5 kg during this period,
‘On physical oxarination, her skin is warm and mist. She has three palpablo, motile thyroid nedulos that measure
between 2 cm and 4 cm nthe largest diameter. There is no lymphadenopathy or bruit, and the thyroid is nontender to
palpation. Hor heart rata is clovated at 144 boats por minuto; tho rata is irregularly iroguler, and a systole murmur can
bbe heard at the let jower sternal border. She has mild edema in both lees. Her reflexes are brisk, and the remainder of
the examination is normal. Laboratory testing reveals hyperthyroidism; the patient hes a low level of thyratrapin and en
eevated level of free thyroxine. A thyroid scan is ordered after the administration of iocine-125.
daspite having a good appatte.
Which one of the following diagnoses is most likely in this patient? م
Several focal areas of increased radioiodine
uptake can be seen, with suppression of
surrounding tissue. Tracer uptake was 28%
within 6 hours after the administration of
jodine-123.
© Drug-induced thyroiditis
O Graves’ disease
O Painful subacute thyroiditis
O Toric adenoma
Toxic multinodular goiter
Y @ Toxic multinodular goiter
صفحه 146:
54-year-old woman presents with increased anxiety, diahea, sweating, and hand tremors thal began 3 months
earlier. She also lost 6.8 kg during this period. She has a history of atrial flrilation and has been treated with
amiodarone for the past 2 years. She has no history of thyroid disease — thyroid functon tests were normal before she
began ireatment with amiodarone
On physical examination, her skin is warm and moist. Her thyroid is normal and nontender and is without nodules. Her
hands show a slight resting tremor. Her heart rate is 80 beats per minute and regular, and there is a systolic murmur at
the left lower sternal border. She has mid edema in both legs. Her reflexas are brisk. and the remainder of the
examination is normal. Laboratory testing reveals hyperthyroidism; the patient has a low level of thyrotropin and an.
olovated lovel of free thyroxine. A thyroid coan is orderod after the administration of iodine-123,
Which one of the following diagnoses is most likely in this patient?
O Drug-induced thyroiditis,
O Gaves' disease
ره
O Toxic adenoma
Toxic multinodular goiter
Roll over scan for more information
‘Tracer uptake was low (<5%) within 6 hours
VY @ Drug-induced thyroiditis | after the Sa metas
صفحه 147:
آقای ۴۳ ساله با پلی اوری-پلی
دیپسی:
۱.نام ساختار های مشخص شده را
؟. با توجه به شكل كدام يك از علل ١
زير جهت توجيه علايم وى محتمل
تر است:
رای
a ۱۰۸۱6۵۳۲۵96016 DI
1 2
پلی دیپسی اولیه ,۲
Y.Central Dl
صفحه 148:
Urinary frequency, nocturia, enuresis
24+h urine volume and osmolarity on unrestricted fd intake
Volume <a0 mig
Osmolarity >800 mosml
تست [Aeon] سس
¥ ¥
Primary polyaipsia Pituitary Di
FIGURE4044 Simplified approach to the differential diagnosis
Of diabetes insipidus. When symptoms suggest ciabetes insipidus
(Dd the syndrome shoul be differentiated from a genitourinary (U)
atmormalty by measuring the 24h urine vakime and osmolarity on
Unrestricted fluid irate If D's confirmed, basal plasma arginine vaso-
pressin (AVE) should ba measured on unrestricted uid intake. f AVP is
normal or elevated (>1 pg/ml, the patient probably has nephrogenic
Di. However. if plasma AVP is low or undetectable, the patent has ether
pituitary Dl ox primary polydipsia,
صفحه 149:
خانم ۵۵ ساله با تنگی نفس پیشرونده احساس خواب آلودگی شدید طی روز :
نام يافته ها (در مجموع ۵ مورد) - تشخیص؟؟؟
HEEL PAD THICKENING
5 و اما درمان انتخابى در صورت
Lateral radiograph of skull reveals enlarged sella with شرایط بالینی مناسب؟؟؟
double flooring, thickened skull vault, pneumosinus
dilatans and prognathism.
صفحه 150:
However, 50% of patients require at least 8 years for GH levels to
be suppressed to <5 j1g/1; this level of GH reduction is achieved in
about 90% of patients after 18 years but represents suboptimal GH
suppression. Patients may require interim medical therapy for several
years before attaining maximal radiation benefits. Most patients also
experience hypothalamic-pituitary damage, leading to gonadotro-
pin, ACTH, and/or TSH deficiency within 10 years of therapy.
In summary, surgery is the preferred primary treatment for
GH-secreting microadenomas (Fig. 403-5). The high frequency of GH
hypersecretion after macroadenoma resection usually necessitates
adjuvant or primary medical therapy for these larger tumors. Patients
unable to receive or respond to unimodal medical treatment may
benefit from combined treatments, or can be offered radiation.
Git RECEPTOR ANTAGONIST
Pegvisomant antagonizes endogenous GH action by blocking
peripheral GH binding to its receptor. Consequently, serum IGF-I
levels are suppressed, reducing the deleterious effects of excess
endogenous GH. Pegvisomant is administered by daily subcutane-
‘us injection (10-20 mg) and normalizes IGF-I in ~70% of patients.
GH levels, however, remain elevated as the drug does not target
the pituitary adenoma. Side effects include reversible liver enzyme
elevation, lipodystrophy, and injection site pain. Tumor size should
be monitored by MRI.
Combined treatment with monthly somatostatin analogues and
weekly or biweekly pegvisomant injections has been used effec-
tively in resistant patients.
DOPAMINE AGONISTS
Bromocriptine and cabergoline may modestly suppress GH secre-
tion in some patients. Very high doses of bromocriptine (220 mg/d)
or cabergoline (0.5 mg/d) are usually required to achieve modest
GH therapeutic efficacy. Combined treatment with octreotide and
cabergoline may induce additive biochemical control compared
with either drug alone.
RADIATION
External radiation therapy or high-energy stereotactic techniques are
used as adjuvant therapy for acromegaly. An advantage of radiation
is that patient compliance with long-term treatment is not required.
Tumor mass is reduced, and GH levels are attenuated over time.
صفحه 151:
خانم ۳۵ ساله دونده با پیوست مزمن و شکستگی فمور هر دو سمت
“Salt-and-pepper” skull
produced by alternating
zones of lucency and
sclerosis. Loss of trabecular
detail in the diploic space
blurs t the:separation
between i inner. and outer
tubules
مس افا
Parameter Guldetine
صفحه 152:
پسر ۲۳ ساله سیگاری و دیابتی با سابقه مثبت خانوادگی بیماری کرونر با حملات مکرر
درد شکم. نام یافته ها و درمان های انتخابی؟؟
صفحه 153:
FAMILIAL HYPERTRIGLYCERIDEMIA (FHTG) FHTG is characterized by ele-
vated fasting TGs without a clear secondary cause, average to below
average LDL-C levels, low HDL-C levels, and a family history of
hypertriglyceridemia. Plasma LDL-C levels are often reduced due to
defective conversion of TG-rich particles to LDL. In contrast to FCHL,
apoB levels are not elevated. The identification of other first-degree
relatives with hypertriglyceridemia is useful in making the diagnosis.
Unlike in FCHL, this condition is not generally associated with a sig-
nificantly increased risk of CHD. However, if the hypertriglyceridemia
is exacerbated by environmental factors, medical conditions, or drugs,
the TGs can rise to a level at which acute pancreatitis is a risk. Indeed,
management of patients with this condition is mostly geared toward
reduction of TGs to prevent pancreatitis.
Individuals with this phenotype generally have reduced lipolysis
of TRLs, although overproduction of VLDL by the liver can also con-
tribute. No single gene has been identified in which mutations cause
this disorder, whereas combinations of gene variants have been shown
to cause this phenotype. A more appropriate term for this condition
might be polygenic hypertriglyceridemia.
صفحه 154:
دختر ۲۱ ساله با آنمی و اختلالات خلقی و عدم تعادل: توصیف دقیق مدالیته و یافته ها- تشخیص
patients with WD (a) T2W axial image showing
ntense signal changes in bilateral putamen and thalami;
(b) TLW axial sequence with hyperintensity of both pallidum
(c) FLAIR axial sequence revealing midbrain tectal plate signal
صفحه 155:
توصیف یافته ها (۲ مورد) 1۹۴
Macronodular liver
cirrhosis, relative sparing
of the caudate lobe, and
moderate splenomegaly.
صفحه 156:
Kayser-Fleischer ring
3 يافته؟؟؟؟! ! !!
شرمنده! فقط محض خنده
عکس اسکلرای نرمال بود
صفحه 157:
Kayser-Fleischer (K-F) rings seen in Wilson's disease is
due to copper deposition in the Descemet's membrane
in the sclero-corneal junction. K-F rings seen here on
blue iris before and after treatment.
صفحه 158:
پسر ۱۱ ساله با سردرد» تاری دید و پلی اوری از ۳ ماه قبل
0۳555 A benign suprasellar tumour arising from
squamous epithelial remnants of Rathke's pouch (from
which the anterior pituitary develops)
* It is the most common paediatric tumour of the
suprasellar cistern
Visual failure (due to optic chiasm
compression) ® headache (secondary to raised intra-
cerebral pressure following obstruction of the foramen
of Monro) » endocrine disturbances (due to pituitary
compression)
* Although it occurs most frequently during childhood,
a further peak is seen during the 6"" decade
‘The tumour tends not ما expand
the pituitary fossa unless it is very large (which is a dif-
ferentiating feature from a pituitary macroadenoma)
Location It is commonly located within the hypotha-
lamic region » it less commonly involves both the
suprasellar and intrasellar regions ® a purely intrasellar
region is rare
CT A cystic and calcified suprasellar tumour (calcifica~
Craniopharyngiomas in two children. (A.B) The first child has a Hon #8 often. present in childhood tumours but less یه
large suprasellar, prepontine and middle cranial fossa tumour 9 doen an cial furacurs)
which is causing considerable mass effect on the brainstem and is 25۳ Although usually cystic, there can also be a ¢olid
۱ . .: or mixed cystic/solid appearance ® solid. components
encasing the basilar artery (arrowheads). There are calcified اقب بو
components (arrows). The cystic components are of higher density © Cystic components: TIWE: slightly hyp
on CT in keeping with proteinacous contents. (due to the protein content) » T2WI
صفحه 159:
خانم 55 ساله دیابتی, تشخیص ؟
Necrobiosis lipoidica diabeticorum is an uncommon dis-
order, accompanying diabetes in predominantly young women. This
usually begins in the pretibial region as an erythematous plaque or
papules that gradually enlarge, darken, and develop irregular margins,
with atrophic centers and central ulceration.
صفحه 160:
خانم ۶۵ ساله با سر درد و
تاری دید پیشرونده :
توصیف؟
تشخیص؟
درمان انتخابی؟
7 ۱ ما Anterior Posterior
صفحه 161:
PHARMACOLOGIC AGENTS APPROVED FOR TREATMENT OF
PAGET'S DISEASE
Dose and Mode
Name of Delivery Normalization of ALP
Zoledronic acid 5 وم V over 15 min 90% of patients at 6 mo
Pamidronate 30mg IV/d over4hon ~50% of patients
3 days
Risedronate 30mg PO/dfor2mo 73% of patients
Alendronate 40 mg PO/d for 6 mo 63% of patients
Tiludronate 800 mg PO daily for 35% of patients
3mo
Etidronate 200-400 mg PO/d x 15% of patients
6mo
Calcitonin (Miacalcin) 100 U SC daily for 6-18 (Reduction of ALP by up
mo (may reduce to50U to 50%)
3 x per wk)
صفحه 162:
خانم 27 ساله با 2 نوبت حمله پانکراتیت در 6 ماه گذشته مراجعه نموده است
صفحه 163:
* Diaguvsis:
۰ بمزروجو) MO lipol vic uctivity iat postkeparia لام
هون وا dePiciewy, if اه تاه the oddiiza oF cored plaiswu.
* Oolevular sequeuviay oP the yours.
و
* Oetry Pat resiriciiva (IS qd) wi امه متمموان طلطیاموه<
* Pick vile
* Ve the wute seticg oP 0-4141)صمه dePiciewy: PRC
۰ ) yeve therupy upprowk: dipoyeur fiparvover
صفحه 164:
صفحه 165:
ی )]
« عومرمز()
* Oorapectaiog oP very high levels oP تسا مج
*“B-quaciPicaiva” by utrarecrPugation (OLOL-C/TC >O.90)
* Lipoproteis electrophoresis (broud B bac)
* Ourtear waaetiz باس مسا
* OO@-bused wethods (apoE yeutppicn): uprx~PC/EO
* Dreakvet
" biPestle
* Clivicaiog or teeukoeat oP previpitaicg Poctors
* Grotics
— اا اراد بر ۰
صفحه 166:
Miscellaneous
صفحه 167:
. Use of HLA-identical grafts eliminates the risk of this con-
dition
. This condition typically occurs 7 to 21 days after transplan-
tation
. Increased age is a risk factor for this condition
. Sex mismatch (female donor, male recipient) is a risk factor
for this condition
. Values on liver function tests are often increased in this con-
dition
37-year-old man received a bone marrow transplant 2 weeks
previously for acute myelogenous leukemia. Two days ago,
itching, diarrhea, pain and numbness in his palms and soles,
and the skin lesions shown here developed. Each of the follow-
ing statements about this condition is true except:
صفحه 168:
Acute Graft-Versus-Host Disease
Answer a
+ Occurs 7 to 21 days after transplantation
+ Donor T cells attack host HLA antigens
‘+ Mean frequency in adults with HLA-identical grafts is 35%
۰ Risk factors include the following:
Increased age
HLA mismatch
Sex mismatch (female donor, male recipient)
Irradiation
Suboptimal immunosuppression
+ Alfects the skin, gastrointestinal tract, and liver
+ Pruritus and pain may be the first sensations, commonly
followed by an erythematous measles-like maculopapular
exanthem. Acral erythematous lesions may develop on,
palms, soles, and ears
‘+ Blistering and exfoliation are common
‘+ Digestive tract involvement may present (from less severe to
more) as nausea, vomiting, anorexia, diarrhea, malabsorption,
abdominal pain, ileus, and ascites
‘+ Values on liver function tests are often increased; jaundice
and hepatomegaly may develop
‘+ Treatment involves intensifying immunosuppression, such
as high-dose corticosteroids, cyclosporine, and cyclophos-
phamide
‘© Severe cases (grade IV) have a high mortality rate (>80%)
صفحه 169:
a. Ehlers-Danlos syndrome
b. Cutis laxa
c. Osteogenesis imperfecta
d. Congenital contractural arachnodactyly
e. Marfan syndrome
‘These are the hand and skin findings of a 26-year-old woman
who complains of early satiety. Her past medical history is
notable for upper gastrointestinal bleeding and rectal prolapse.
‘What is the diagnosis?
صفحه 170:
Ehlers-Danlos Syndrome
Answer: a
Ehlers-Danlos syndrome is characterized by highly elastic
connective tissue
Many forms (up to 15) of Ehlers-Danlos syndrome exist
The autosomal-dominant forms of the disease account for
90% of reported cases
Patients have hyperextensible and lax joints that are prone
to dislocation
Patients with skin manifestations have hyperextensible, fragile
skin that heals poorly, characteristically forming wide, thin,
“fish-mouth” scars. The skin may have a velvety texture
Patients are predisposed to the following:
Gastrointestinal motility disorders
Visceral diverticulosis
Mitral valve prolapse (up to 50% of patients)
Dilatation of the aortic root
Pes planus
Scoliosis
Degenerative arthritis
Pneumothorax
Dilatation of the pulmonary artery
Angina
صفحه 171:
عتانطاح5 .
b. Cystic medial necrosis
c. Rheumatoid arthritis
d. Hypertension
. Atherosclerosis
p
°
A 54-year-old woman presents with nonreproducible upper
back discomfort and dysphagia. She has a history of coronary
artery disease and abdominal aortic aneurysm repair 6 years
ago. Thoracic computed tomography scans are shown. In the
United States, which one of the following risk factors is most
commonly associated with this condition?
صفحه 172:
Descending Thoracic Aortic Aneurysm
Answer: e
۰ Aortic aneurysms are classified by location (abdominal or
thoracic) and shape (fusiform or saccular, as in this case)
* Thoracic aneurysms are further classified as ascending or
descending, based on their location proximal or distal to the
aortic arch
* Most often aortic aneurysms are asymptomatic
* Compression of adjacent structures (the esophagus in this,
case) may result in symptoms. Leakage of blood from the
aneurysm may cause acute pain and may be a sign of
impending rupture
* Acute rupture without warning is the most common cause
of symptoms. Therefore, clinical suspicion for the diagnosis
should be high
+ When one aneurysm is detected, the patient should be
screened for the presence of other occult aneurysms
* Size correlates with risk of rupture, but not as exactly as for
abdominal aortic aneurysms. Nonetheless, most authors
favor surgical management for thoracic aortic aneurysms
that are more than 6 cm in diameter
* Atherosclerosis is the most commonly associated condition.
Others include hypertension, giant cell arteritis, syphilis,
Ehlers-Danlos syndrome, rheumatoid arthritis, trauma,
cystic medial necrosis, and Marfan syndrome
صفحه 173:
a. Exposure to sunlight
b. Diffuse muscular pain
c. Proctoscopic examination
d. Fat aspirate biopsy
e. Hypertension
57-year-old man with known lymphoproliferative
disorder presents with gastrointestinal bleeding and
diarrhea. Periorbital purpura may occur in this patient
and is associated with which one of the following?
صفحه 174:
Amyloidosis With Pulmonary and
Gastrointestinal Involvement
Answer: ¢
‘+ Amyloidosis may have multiorgan involvement, including
liver, kidney, gastrointestinal tract, and heart. Up to 90% of,
patients with primary amyloidosis have cardiac dysfunction
+ The liver may become infiltrated in up to 25% of cases and
indicates extensive involvement and a poor prognosis,
‘+ Manifestations result from deposition of an amorphous,
insoluble protein-polysaccharide complex in the tissue
‘+ There are several types of amyloid, and their classification
is based on the type of protein fibrillar deposition:
Type AA is associated with reactive systemic amyloidosis
and is found in hereditary ot acquired chronic inflam-
matory disease
Type AL is associated with systemic amyloidosis and
is found in multiple myeloma and monoclonal gam-
mopathies|
‘* Gastrointestinal manifestations include diarrhea, megacolon,
and fecal incontinence
‘+ Periorbital purpura may occur after proctoscopic examination
‘+ Fat aspiration is confirmatory of the diagnosis in up to 80%
of patients. Rectal biopsy confirms the diagnosis in up to
75% of patients
‘+ Congo red stain of histologic specimens may reveal a char-
acteristic apple-green birefringence under crossed polarized
light. This remains the standard test for diagnosis
صفحه 175:
a. Liver
b. Kidneys
c. Heart
d. Lungs
e. Pancreas
The nails shown here are those of a 56-year-old ill-appearing
woman. Which one of the following organ systems has the
hicheet Hkohhand of heine diceaced?
صفحه 176:
Yellow Nail Syndrome
Answer: d
* Classic triad:
Yellow nails
Extremity lymphedema
Pleural effusions
+ Nail findings:
Discoloration: yellow to yellow-green or brownish yellow
Slow growth
Onycholysis
+ Pulmonary associations (not in 100%):
Pleural effusion (35%-40%; lymphocyte predominant,
often bilateral, one-third are recurrent)
Restrictive and obstructive lung defects
Bronchiectasis (20%)
* Systemic associations (not in 100%):
Rhinosinusitis
Chronic edema of lower extremities
Breast edema
Raynaud’s phenomenon
Pericardial effusion
+ Lymphatic insufficiency is common and results in the edema
* Nail changes may be reversible and do not necessarily cor
relate with other manifestations of the syndrome (such as
pulmonary disease, edema)
© No treatment has been proved effective
صفحه 177:
a. Systemic lupus erythematosus
b. Defective porphyrin metabolism
cc. Autoimmune reaction to self-antigens in the skin
d. Autosomal recessive defect in DNA excision repair mechanism.
e. A vertically transmitted hepatitis virus
In this 15-year-old girl, these lesions developed in sun-exposed.
areas. She is exquisitely sensitive to sunburn. What is the cause
of this rare disorder?
صفحه 178:
Xeroderma Pigmentosum
Answer: d
‘+ Autosomal recessive disorder found in all racial groups
‘+ Multiple mutations have been identified, but all involve
defects in the excision repair mechanism of damaged DNA
in skin exposed to ultraviolet light (nucleotide excision
repair)
‘Skin findings:
Actinic keratoses
Cutaneous melanoma
Progressive atrophy
Irregular pigmentation
Telangiectases
Basal cell carcinoma
Squamous cell carcinoma
‘+ Patients are very sensitive to sun exposure, as evidenced by
reports of tongue tumors
‘Skin cancers develop at an early age (median, 8 years)
‘+ Most patients die of these malignancies by the third decade
of life
‘+ Risk for skin cancer is 2,000- to 10,000-fold greater in patients
with xeroderma pigmentosum than in age-matched controls
‘* Ophthalmic and neurologic disorders also occur
* Diagnosis is established with the fibroblast survival test after
exposure to ultraviolet light